Part 2 Units 2-15(Most tested section) Questions

Réussis tes devoirs et examens dès maintenant avec Quizwiz!

B) Short straddle Explanation If the spot price does not move, at least one option—perhaps both—will expire unexercised. The other positions will not yield profit without price movement. Note that a short straddle carries unlimited loss potential. LO 10.f

A speculative investor believes the market in Japanese yen will remain stable for several months. Which of the following positions might allow the investor to take advantage of a lack of movement in the exchange rate between the yen and the U.S. dollar? A) Call debit spread B) Short straddle C) Put debit spread D) Long straddle

A) issue bonds Explanation A mutual fund may not issue any senior securities, although it may purchase almost any type of security for its portfolio. LO 8.b

An open-end investment company may do all of the following except A) issue bonds. B) redeem shares. C) continuously offer shares. D) borrow money.

B) the procedures for the annual election of general partners Explanation Limited partners have limited liability. General partners have unlimited liability. Only in specific situations can the limited partners elect a new general partner. Such situations would include the resignation, death, incapacity, or removal of the general partner. LO 11.d

B) the procedures for the annual election of general partners Explanation Limited partners have limited liability. General partners have unlimited liability. Only in specific situations can the limited partners elect a new general partner. Such situations would include the resignation, death, incapacity, or removal of the general partner. LO 11.d

A) Specialized Explanation A specialized or sector fund invests 25% or more of its assets in a particular region or industry. LO 8.g

If an investor wants to invest in the electronics industry but does not want to limit his investments to one or two companies, which type of fund would be most suitable? A) Specialized B) Hedge C) Money market D) Bond

D) Odd lot Explanation Odd lots are usually traded by small investors; some analysts believe small investors are generally wrong. LO 13.e

Proponents of which of the following technical theories assume that small investors are usually wrong? A) Breadth of market B) Volume of trading C) Short interest D) Odd lot

D) 8.5% of the offering price. Explanation Open-end investment companies (mutual funds) are limited to a maximum sales charge of 8.5% of the offering price. LO 8.d

Under the Conduct Rules, the maximum sales charge on any transaction involving an open-end investment company share is A) 8.5% of the net asset value. B) 9% of the offering price. C) 9% of the net asset value. D) 8.5% of the offering price.

C) Only the principal Explanation Neither the customer's signature nor the registered representative's signature is required to open a cash account. A principal must review and accept the new account by signing the form. LO 1.d

f a customer wishes to open a cash account, who must sign the new account form? A) Only the customer B) Only the registered representative C) Only the principal D) The customer, the registered representative, and the principal

D) Index options Explanation Systematic risk is the market risk applied to the value of an entire portfolio instead of just one stock. Therefore, an investor would use index options to protect against systematic risk. LO 10.g

,Which of the following could be used to protect an equity portfolio against systematic risk? A) Interest rate option B) Stock options C) Currency options D) Index options

D) II and IV Explanation According to FINRA rules, when an employee of a member firm opens an account with another member broker-dealer, duplicate confirmations and account statements must be sent to the employer by the broker-dealer establishing the account when the employer requests it to do so. An officer of a broker-dealer is considered an employee. LO 1.d

According to FINRA rules, duplicate confirmations of transactions must be sent to an account owner's employer—if requested to do so by the employer—whenever establishing a margin account for a bank officer. an employee of another broker-dealer. an independent insurance agent. an officer of another broker-dealer. A) I and II B) III and IV C) I and III D) II and IV

B) 4 years Explanation The Municipal Securities Rulemaking Board requires firms to retain abstracts of official statements for four years—the same as all pieces intended to communicate with the public. LO 6.h

An abstract of a municipal securities issue official statement must be maintained on file for how long? A) 12 months B) 4 years C) There is no requirement to file abstracts of official statements. D) 5 years

C) $54.50 Explanation To determine the breakeven point, net the option premiums (8 − 3.50 = 4.50). For a call spread, add the netted premiums to the lower strike price (50 + 4.50 = 54.50). LO 10.h

An investor buys 1 XYZ Nov 50 call at 8 and sells 1 XYZ Nov 60 call at 3.50. At what stock price will the investor break even? A) $60.00 B) $63.50 C) $54.50 D) $50.00

B) Roth IRA Explanation Given the investor's current age (40), a safe assumption is that the investor will have owned the Roth IRA for at least five years before any distributions would be taken. Roth IRAs allow for tax-free distributions when owned for five years and the recipient is age 59½ or older, while traditional IRAs do not. Municipal bonds offer tax-free interest but do not grow tax deferred, and variable annuities should be used only after contributions to employer-sponsored plans and IRAs are maxed out. LO 1.g

An investor, age 40, earns $65,000 annually and contributes 3.5% to his employer's 401(k) plan. With the 401(k), his only retirement savings, he wants to do more for retirement and hopes to invest in such a way as to have some tax-free income when he takes distributions later in life. Which of the following is the most suitable given the investor's goals and objectives? A) Traditional IRA B) Roth IRA C) Municipal bonds D) A nonqualified variable annuity

B) hedge a long stock position with a short put. Explanation Writing a put does not reduce the risk of a long stock position. The short put creates an obligation to purchase additional shares if the put is exercised (which will happen if the stock falls). A long call is an effective hedge against a short stock position. LO 10.d

An option investor might do all of the following except A) hedge a short stock position with a long call. B) hedge a long stock position with a short put. C) make an opening sale. D) make a closing purchase.

D) coterminous debt. Explanation In the context of municipal securities, the term coterminous refers to two or more taxing agencies that share the same geographic boundaries and are able to issue debt separately. Overlapping debt occurs when two or more issuers are taxing the same property to service their respective debt. LO 6.b

Another term for municipal overlapping debt is A) double-barreled debt. B) defeased debt. C) refunded debt. D) coterminous debt.

D) 50% Explanation When a U.S. corporation receives dividends from another U.S. corporation it has invested in, 50% of the dividends received are excluded from taxation (tax free). Therefore, 50% of the remaining dividends received are taxable. LO 13.g

BAKE-ALL, a U.S. manufacturing corporation, has purchased shares of stock in RE-FORM, a U.S. corporation that refines raw materials. RE-FORM pays a dividend to its shareholders. For BAKE-ALL corporation, taxes will be due on what percentage of the dividends received from RE-FORM? A) 0% (all dividends received are tax free) B) 70% C) 100% D) 50%

A) the feasibility study Explanation Because most revenue bonds depend on cash flow from a facility or project, a feasibility study is done to determine its justification. LO 6.b

Before issuing a revenue bond, an issuer will engage various consultants to prepare a report detailing the need for a particular project. This is generally called A) the feasibility study. B) the consultant's report. C) the economic study. D) the revenue report.

A) the branch manager initially as long as it is subsequently approved by a principal who has a Series 10 registration or is a Registered Options Principal (ROP) Explanation In general, an options account will be approved by a Limited Principal with a Series 10 license or a Registered Options Principal (ROP). If the branch office manager is not a Registered Options Principal or a Limited Principal—General Securities Sales Supervisor, account approval or disapproval shall, within ten (10) business days, be submitted to and approved or disapproved by a Registered Options Principal or a Limited Principal—General Securities Sales Supervisor. LO 10.j

Customers seeking to open an options account may have the account approved by A) the branch manager initially as long as it is subsequently approved by a principal who has a Series 10 registration or is a Registered Options Principal (ROP). B) the branch manager, only as long as they have a Series 10 registration or are a Registered Options Principal (ROP). C) the registered investment adviser managing the client's portfolio. D) the registered representative handling the account.

D) General partnership Explanation Limited partnerships would not work because the other investors have limited say in how the enterprise is run. C corporations do not provide favorable tax treatment of gains or losses. Although an S corporation appears to be the right answer, only U.S. citizens or resident aliens can own one. LO 1.c

Dale Wells, a British citizen temporarily working in the United States, wants to form a business venture with other investors. Wells is looking for favorable tax treatment of earnings and losses. Wells also wants to limit the number of investors but is willing to share control of the enterprise with others to attract them. What business form would you advise? A) C corporation B) S corporation C) Limited partnership D) General partnership

C) the total of interest and principal payable by the issuer plus any amount required to be deposited into a sinking fund Explanation Debt service is the total of interest and principal payable by the issuer plus any amount required to be deposited into a sinking or surplus fund. LO 6.b

Debt service is best described as A) total of the direct debt of a municipality and the debt of its political subdivisions. B) services provided by the paying agent for a bond issue. C) the total of interest and principal payable by the issuer plus any amount required to be deposited into a sinking fund. D) net interest on a new issue of a municipal bond.

A) $0.01 Explanation At expiration, unless the customer instructs not to, equity options that are in the money by $0.01 or more will be automatically exercised. This rule is applicable to both retail and institutional customers. LO 10.j

Equity options that are in the money by what amount will be automatically exercised at expiration, barring any other instructions from the customer? A) $0.01 B) $0.10 C) $1.00 D) $0.05

D) the recommendation made would be suitable for at least some customers. Explanation This question refers to the three specific obligations under Rule 2111. Those three are reasonable-basis suitability, customer-specific suitability, and quantitative suitability. Complying with the first of the three means the registered representative has to have a reasonable basis to believe that a recommendation is suitable for at least some investors. Control relationships must always be disclosed, but that is not part of the three obligations. Compensation may have to be disclosed, but, once again, that is not part of the three obligations. Be sure to focus on answering the question being asked. LO 2.f

FINRA Rule 2111 places three obligations on members when determining if a specific recommendation to a customer is suitable. FINRA's suitability rules would likely find a registered representative is not in violation of complying with those three if A) proper disclosures were made of the representative's compensation received. B) control relationships were disclosed. C) the recommendation was profitable for the investors. D) the recommendation made would be suitable for at least some customers. Explanation This question refers to the three specific obligations under Rule 2111. Those three are reasonable-basis suitability, customer-specific suitability, and quantitative suitability. Complying with the first of the three means the registered representative has to have a reasonable basis to believe that a recommendation is suitable for at least some investors. Control relationships must always be disclosed, but that is not part of the three obligations. Compensation may have to be disclosed, but, once again, that is not part of the three obligations. Be sure to focus on answering the question being asked. LO 2.f

C) stay the same Explanation In a long straddle, you are buying a put and a call. Holders can only profit if the stock moves farther away from the strike price than the total of the premiums paid. LO 10.f

Holders of long straddles would like the underlying stock to do all of the following except A) fluctuate. B) go down. C) stay the same. D) go up.

D) net direct debt plus overlapping debt. Explanation Net overall debt of a municipality is defined as net direct debt plus overlapping debt. LO 6.b

Net overall debt of a municipality is A) funded debt minus overlapping debt. B) net direct debt minus overlapping debt. C) funded debt plus overlapping debt. D) net direct debt plus overlapping debt.

B) principal repayment on partnership debt. Explanation Losses occur when expenses exceed revenues. Principal repayments are not an expense. Interest on debt is a deductible expense. Natural resources deplete and the depletion allowance is an expense similar in concept to depreciation, another expense. From a personal standpoint, compare this to your home mortgage−the interest is a deductible expense, but the portion representing payment of principal is not. LO 11.f

One of your customers owns a limited partnership interest in an oil and gas drilling program. The program was successful in finding oil and is expected to operate at a loss for the next year. The loss flowing through to the limited partner is generated by all of these except A) accelerated depreciation taken on the drilling equipment. B) principal repayment on partnership debt. C) interest payments on partnership debt. D) depletion on the sale of oil removed from the ground.

A) the separate account performance depends on the performance of the selected subaccounts Explanation Some insurance company separate accounts have dozens of different subaccounts. These subaccounts range from highly aggressive to highly conservative. It is the performance of the specific subaccounts selected by the investor that determines the value of their accumulation unit. The actuaries get involved when determining the payout because that depends on life expectancy. Similarly, the AIR comes into play only during the payout phase. LO 9.c

One of your customers owns a variable annuity. When asking about how the performance of the separate account is measured, you would respond that A) the separate account performance depends on the performance of the selected subaccounts. B) the separate account performance is the same for all subaccounts. C) the insurance company's actuaries compute the separate account performance. D) the primary determinant is the assumed interest rate (AIR).

C) I and IV Explanation Investment policy, track record, portfolio, and sales load should all be researched when assessing a fund. The identity of the custodian bank for the fund or the number of shares outstanding does not bear on its performance or suitability. LO 8.j

Potential investment company clients should be advised to investigate a fund by looking at which of the following? Investment policy Number of shares outstanding Custodian bank. Portfolio A) I and III B) II and III C) I and IV D) II and IV

A) Real estate investment trusts (REITs) Explanation REITs allow for the direct pass-through of income, but not losses. The other choices are forms of business that allow for pass-through of income and losses. LO 11.b

Programs allowing for the direct pass-through of losses and income to investors include all of the following except A) real estate investment trusts (REITs). B) new-construction real estate direct participation programs. C) oil and gas drilling direct participation programs. D) S corporations.

D) tax free. Explanation If a withdrawal from a Roth IRA is a qualified distribution, the withdrawal is tax free. A qualified distribution is made after a five-year holding period and after the taxpayer has reached age 59½. LO 1.g

Qualified distributions from Roth IRAs are A) tax deferred. B) taxable only to the extent of earnings. C) 100% taxable. D) tax free.

A) preferred REITs Explanation While some corporations may issue preferred stock, there is no such term as a preferred REIT. Equity REITs take ownership positions in real estate properties. Mortgage REITs make loans to real estate properties. Hybrid REITs do both. LO 11.a

Real estate investment trusts (REITs) offer investors an opportunity to pool their money with others to receive professional management of real estate. Generally available types of REITs include all of the following except A) preferred REITs. B) hybrid REITs. C) equity REITs. D) mortgage REITs.

B) a registered representative (RR) accepting unsolicited orders to exercise options. Explanation An individual investor or a group of investors acting in concert must observe position and exercise limits. These limits apply to an individual adviser acting for a group of discretionary accounts and to an individual who has accounts with several firms. Acting in concert does not apply to an RR simply accepting exercise order instructions from customers. LO 10.j

Regarding rules addressing acting in concert, each of the following must observe position and exercise limits except A) an individual with accounts at several brokerage firms. B) a registered representative (RR) accepting unsolicited orders to exercise options. C) two or more individuals who have an agreement to act together. D) an investment adviser placing exercise orders for his discretionary accounts.

A) permissible if performed in compliance with the 5% policy Explanation The 5% policy applies to all types of nonexempt secondary market transactions, including riskless and simultaneous transactions. An example of a riskless or simultaneous transaction is one where a member buys a security to fill an order for the same security previously received from a customer. LO 13.g

Riskless or simultaneous transactions by a broker-dealer are A) permissible if performed in compliance with the 5% policy. B) permissible only if there is a profit for a customer. C) not permissible under any circumstances. D) permissible only in new issue underwritings.

C) The VIX Index Explanation The VIX Index, sometimes called the "fear" index, is considered to be the most accurate indicator of anticipated market volatility. LO 10.g

Securities traders would consider which of the following to be the most relevant indicator of expected market volatility? A) The S&P 500 Index B) The market's beta C) The VIX Index D) The advance/decline ratio

C) a 2x ETF Explanation Leveraged funds (2x or 3x) are considered the investment companies carrying the highest risk. Although high-yield bonds, at least on an individual basis, are considered high risk, the diversification and professional management of the mutual fund reduce (do not eliminate) the risk. LO 8.i

The diversification and professional management offered by many investment companies tends to lower the investor's risk. That does not mean elimination of risk. Of the following, it is likely the greatest risk would be investing in A) a high-yield bond mutual fund. B) an equity UIT. C) a 2x ETF. D) an broad market index fund.

C) the issuer has the authority to issue bonds that are legal, valid, and enforceable obligations of the issuer Explanation Bond counsel attests that, to the best of its knowledge, the issuer has the legal right to issue the securities in question. In the case of tax-exempt bonds, the interest the issuer will pay on the bonds is exempt from federal taxation, and the bonds are exempt from federal registration requirements. The legal opinion does not go to the issue's marketability—or safety—debt service requirements. LO 6.a

The unqualified legal opinion on a municipal bond states that A) the bond has passed the additional bonds test (parity test). B) the issuer is creditworthy. C) the issuer has the authority to issue bonds that are legal, valid, and enforceable obligations of the issuer. D) the bond is marketable.

C) April 15 of the following year Explanation Contributions to IRAs can be made up to April 15 of the year following the year for which the contribution is being made. LO 1.g

What is the latest date that an IRA participant may make an IRA deposit for the current year? A) December 31 of the current year B) April 15 of the current year C) April 15 of the following year D) July 15 of the following year, if extensions have been filed

D) the difference in the expected return of the portfolio, given the portfolio's beta, and the actual return the portfolio achieved. Explanation Alpha is the difference in the expected return of the portfolio, given the portfolio's beta and the actual return the portfolio achieved. The higher the alpha, the better the portfolio has done in achieving excess or abnormal returns. LO 13.b

n portfolio theory, the alpha of a security or a portfolio is A) the portfolio's average return divided by the security's beta. B) a measure of the variance in returns of a portfolio divided by its average return. C) the risk of the portfolio associated with the macroeconomic factors that affect all risky assets. D) the difference in the expected return of the portfolio, given the portfolio's beta, and the actual return the portfolio achieved.

D) 4.7% Explanation The basis of a municipal bond is its yield to maturity (YTM). The terms are used interchangeably. Therefore, in this question, the basis is the 4.7% stated as the YTM. There will be some questions on the exam similar to this where no math is involved. Just be sure to know your definitions. LO 6.a

A 4% municipal bond maturing in 2040 has a current yield of 4.4% with a yield to maturity of 4.7%. What is the basis of this bond? A) 4.4% B) 4.0% C) 4.5% D) 4.7%

B) $920 Explanation To determine the parity price of the bond, first find the number of shares the debenture is convertible into (conversion ratio) by dividing par value by the conversion price ($1,000 / $25 = 40 shares). Next, multiply the current price of the common by the conversion ratio. The result is the parity price of the bond (40 shares × $23 = $920). LO 5.d

A 7% convertible debenture is selling at 101, and it is convertible into the common stock of the same corporation at $25. The common stock is currently trading at $23. What is the parity price of the debenture? A) $929 B) $920 C) $850 D) $910

C) $2,050,000. Explanation Remember the 50% dividend exclusion available to C corporations. All of the $2 million of pretax income is taxable along with half of the $100,000 in dividends. The interest is included (as the exam often does) as an extra number to confuse. Pretax income is always after all expenses including interest. LO 13.h

A C corporation's income statement renders the following information: pretax income: $2,000,000; dividends from preferred stock issued by other corporations: $100,000; interest paid on outstanding debentures: $200,000. This corporation has taxable income of A) $1,850,000. B) $1,900,000. C) $2,050,000. D) $2,100,000.

C) a negative response letter Explanation An example of a bulk transfer is the member firm deciding to switch money market funds used for sweeps of customer credit balances. A negative response letter is one where the customer's agreement is assumed unless responding negatively to the change. LO 1.e

A FINRA member firm making a bulk transfer of customers' assets would most likely give notification through A) FINRA's Central Registration Depository (CRD). B) a positive response letter. C) a negative response letter. D) a broadly circulated publication such as the Wall Street Journal.

D) Buy calls on Japanese yen Explanation The U.S. company is concerned that the value of the Japanese yen will rise. Therefore, the company should buy calls on the yen to lock in the lowest possible price to buy yen for payment of the contract. Importers buy calls to hedge. LO 10.g

A U.S. company that sells stereo equipment places an order for Japanese stereo components for its inventory. Payment must be made in Japanese yen in three months. The U.S. company thinks that the U.S. dollar may weaken against the yen. Which of the following foreign currency option transactions would best protect the U.S. company from a weakening of the U.S. dollar against the yen? A) Buy puts on Japanese yen B) Sell calls on Japanese yen C) Sell puts on Japanese yen D) Buy calls on Japanese yen

B) Defined benefit Explanation A defined benefit retirement plan establishes, in advance, the payout to be received by the retiree. The formula is based on earnings and years of service. LO 1.h

A retiree is paid an annual amount equal to 30% of the average of his last five years' salary. Which of the following retirement plans offers this type of payment? A) Profit-sharing B) Defined benefit C) Deferred compensation D) Defined contribution

B) domestic corporate common and preferred stock Explanation Corporate ownership of another company's stock allows the investor to exclude 50% of the dividends from taxation. LO 13.g

A successful chain of retail stores in the maximum corporate tax bracket may exclude from taxation 50% of income earned on investments in A) municipal bonds from the same state in which the corporation is located. B) domestic corporate common and preferred stock. C) government and agency securities. D) industrial development bonds.

B) January Explanation Member firms must provide an IRS Form 1099 to the primary account holder of all the interest and dividends credited to the account in January. This form is used in the customer's tax return preparation. LO 15.b

A summary statement of all interest and dividends credited to a customer's account must be sent to the primary accountholder each year in A) December. B) January. C) April. D) July.

A) declaration of increased dividends. Explanation A technical analyst is interested in statistics about market or price performance, not the fundamental factors, the market, or the company's dividend policy. Technical analysts are interested in trading volume as a market statistic, new highs and lows, and open short positions, which could indicate future buying potential in the security. LO 13.e

A technical analyst is least concerned with A) declaration of increased dividends. B) trading volume. C) open short positions. D) new highs and lows.

B) support levels. Technical analysts care about price and volume trends in the marketplace, such as support levels. A corporation's earnings, P/E ratio, and current ratio would be of most interest to a fundamental analyst who reviews a company's financial statements in more detail. LO 13.e

A technical analyst would be most interested in A) current ratio. B) support levels. C) price-to-earnings (P/E) ratio. D) corporate earnings.

B) An increase in the short interest Explanation As the short interest increases, it is a bullish signal to technicians. Those short sales are going to have to be covered one day and that buying pressure is bullish on the stock. A head and shoulders top is bearish - it signifies that the market has reached a top. Odd-lot purchasers (the little guy) are always buying and selling at the wrong time. When they are buying more than selling, it is a sign that the market price of the stock is soon going to fall. When the advances outnumber the decliners, it is bullish. When it is reversed, as in the answer choice, it is bearish. LO 13.e

A technical analyst would find which of the following to be a bullish indicator? A) More declines than advances B) An increase in the short interest C) A head and shoulders top D) More odd-lot purchases than sales

B) $124,500.000 Explanation To calculate the working capital, we subtract the current liabilities from the current assets. In this case, current assets total $196,000,000 and the sum of the current liabilities is $71,500,000. The difference between those two numbers is $124,500,000 LO 13.d

A) $216,500,000 B) $124,500.000 C) $54,500,000 D) $212,000,000

B) Maximum gain = $475; maximum loss = $8,025; breakeven point = $80.25 per share. Explanation An option writer's maximum gain is generally limited to the premium (the credit to the account) received. In this case, that is 4¾ points or $475. To determine maximum loss, first think what strategy a put writer is following. Short puts are neutral to bullish! The investor will therefore lose when the market is bearish in the extreme, reaching zero. The maximum loss is the entire difference between the strike price and zero, offset (reduced) by the premium received (strike price minus premium). Breakeven follows the put-down rule. Subtract the $475 premium from the $85 strike price to get $80.25. LO 10.h

An investor writes an EFG Dec 85 put for 4¾ points. What is the investor's maximum gain, maximum loss, and breakeven point? A) Maximum gain = $8,025; maximum loss = $475; breakeven point = $80.25 per share. B) Maximum gain = $475; maximum loss = $8,025; breakeven point = $80.25 per share. C) Maximum gain = $475; maximum loss = $8,025; breakeven point = $89.25 per share. D) Maximum gain = $475, maximum loss is unlimited; breakeven point = $80.25 per share.

A) A time spread Explanation Time spreads, also called calendar or horizontal spreads, consist of two options of the same type with the same strike price, but different expiration months. The strategy expects the market to stay relatively level. The profit arises from the time decay of the later expiration date. A long straddle is profitable only if there is market movement. The same is true with the long call - the market price must go up. A debit put spread is a bearish strategy, so this strategy requires the market price to decline. LO 10.e

An options investor wishing to follow a market-neutral strategy would be most likely to find which of the following most appropriate? A) A time spread B) A debit put spread C) A long broad index call D) A long straddle

A) rise. Explanation Preferred stock is interest rate sensitive. As rates fall, prices of preferred stocks tend to rise, and vice versa. LO 3.e

As interest rates fall, prices of straight preferred stock will A) rise. B) become volatile. C) fall. D) remain unaffected.

A) a higher rating and greater marketability Explanation Advance or prerefunding is refinancing an existing municipal bond issue before its maturity or call date by using money from the sale of a new bond issue. Because the proceeds of the new issue are placed into special U.S. government securities, the rating is automatically at the top. The higher rating increases the marketability. The current bond still exists until the specified call date. As such, the coupon has not changed. There are no taxes to be saved. LO 6.d

Benefits of a municipal bond advance refunding include A) a higher rating and greater marketability. B) a decrease to the issuer's current interest cost. C) tax savings. D) a higher rating and lower coupon rate.

A) I and III Explanation If the customer elects to receive distributions in cash while other investors purchase shares through reinvestment, the customer's proportional interest in the fund will decline. The option to have distributions automatically reinvested allows those purchases to be made at NAV, but a purchase made later would be made at the public offering price like any other new purchase. LO 8.f

Customer A and Customer B both have an open account in a mutual fund that charges a front-end load. Customer A has decided to receive all distributions in cash, while Customer B automatically reinvests all distributions. How do their decisions affect their investments? Receiving cash distributions may reduce Customer A's proportional interest in the fund. Customer A may use the cash distributions to purchase shares later at net asset value (NAV). Customer B's reinvestments purchase additional shares at NAV rather than at the offering price. Due to compounding, Customer B's principal will be at greater risk. A) I and III B) II and III C) I and IV D) II and IV

C) short 1 ABC Jan 50 call and short one ABC Jan 50 put Explanation When trading options, there is one way in which to have a potentially unlimited loss. That is the uncovered (naked) call. When a call option is written without a corresponding long position in the underlying, the writer loses when the price goes up. Because there is theoretically no limit as to how high a stock's price can go, the potential loss is unlimited. In this short straddle position, it is the short call that creates this possibility. With a short put, the lowest a stock's price can go is to zero. With a 50 put, that is a maximum loss of $50 less the premium received. The maximum loss on any long position, stock or option, is what the investor paid for it. LO 10.h

Customers will have a potentially unlimited loss if they are A) short 1 ABC Jan 50 put and long 100 shares of ABC stock. B) long 100 shares of ABC stock and short 1 ABC 50 call. C) short 1 ABC Jan 50 call and short one ABC Jan 50 put. D) long 1 ABC Jan 50 put and long 100 shares of ABC stock.

D) 50% into an S&P 500 index fund (or ETF), 30% into an international index fund and 20% into an investment-grade bond fund Explanation Index funds (or ETFs) are appropriate investment vehicles for investors who believe that active management does not produce returns above the cost. Investing in the S&P 500 would give him returns comparable to what he has enjoyed in the past. However, in recognition of his advancing age, it would appear prudent to diversify by placing some of the money into the international sphere and a portion into the safety of high-grade debt securities. LO 8.g

Gerry Logan has been managing his own securities portfolio for the past 15 years. His returns have been about the same as the S&P 500, and as he gets older, he does not want to have to spend the time and effort to keep up the performance. Logan is currently 55 years old and has sufficient discretionary income and savings that enable him to take moderate risks. He is of the belief that his own experience proves that you can't beat the market. Which of the following would you suggest for him? A) 100% into an S&P 500 index fund (or ETF) B) 50% into an S&P 500 index fund (or ETF), 50% into an investment-grade bond fund C) 50% into an S&P 500 index fund (or ETF), 50% into a money market mutual fund D) 50% into an S&P 500 index fund (or ETF), 30% into an international index fund and 20% into an investment-grade bond fund

C) II and III Explanation Because the customer already held the stock long term when he purchased the put, he was not trying to stretch a short-term gain into a long-term gain. There is no effect on his established holding period of 15 months. Whenever a put is exercised, the stock's sale price (exercise price) is reduced by the premium paid for buying the put. LO 10.i

Having held 100 shares of GHI stock for 15 months, a customer purchases 1 GHI Jan 50 put in December. If the put is exercised before the expiration date and the long stock is delivered, which of the following statements are true? The premium is added to the sale proceeds. The premium is deducted from the sale proceeds. Any gain is long term. Any gain is short term. A) I and III B) II and IV C) II and III D) I and IV

D) One Explanation Exercised stock index options settle on the next business day. LO 10.a

How many business days after an index option is exercised should a cash settlement occur? A) Two B) Five C) Three D) One

B) Buy pound puts Explanation If the company wants to protect its investment, it has to protect against the payment going down in value relative to the cost. To protect against the value of the payment going down in relation to the cost, the company would buy puts on the payment currency. To offset the cost of the puts, the company will also sell calls. The calls will be covered by the ownership of the actual currency. Remember the acronym EPIC: Exporters buy Puts and Importers buy Calls. LO 10.g

IBM sold computers to a Soho retailer and agreed to accept payment of 10 million British pounds in 65 days. In which of the following ways could the company protect the payment against adverse foreign currency fluctuations? A) Buy U.S. dollar puts B) Buy pound puts C) Buy pound calls D) Buy U.S. dollar calls

B) I, III, II, IV. Explanation The liquidation order is as follows: the IRS (and other government agencies), secured debt holders, unsecured debt holders and general creditors, holders of subordinated debt, preferred stockholders, and common stockholders. LO 5.b

If LMN, Inc., has filed for bankruptcy, in what order would interested parties be paid? Holders of secured debt Holders of subordinated debentures General creditors Preferred stockholders A) I, II, III, IV. B) I, III, II, IV. C) IV, I, II, III. D) III, I, II, IV.

A) continue to rise. Explanation A rising advance/decline line indicates that more stocks are rising in price than falling. A rising advance/decline line is a bullish indicator. LO 13.e

If a chart indicates that both the Dow Jones Industrial Average and the advance/decline line have been increasing since January, and the advance/decline line continues to rise, the market should A) continue to rise. B) turn down sharply. C) not change. D) turn down moderately.

A) U.S. territories Explanation Federal income taxes are not currently imposed on interest earned from bonds issued by U.S. territories and possessions (e.g., Puerto Rico, Guam, and the U.S. Virgin Islands). LO 7.e

Municipal securities issued by which of the following are triple tax exempt? A) U.S. territories B) Public authorities C) New York City D) Hawaii

D) $0 Explanation To open an IRA, a person needs earned income. Income from rental real estate is passive income, while income from a trust fund is portfolio income. This customer has no earned income. LO 1.g

One of your customers, age 52, wishes to open an IRA. His annual income is more than $200,000 and consists entirely of income from rental real estate and income from a trust fund. What amount may your customer contribute to his IRA this year? A) $5,500 B) $6.000 C) $7,000 D) $0

D) A real estate investment trust (REIT) Explanation DPPs include any form of business that allows for the direct pass-through of tax consequences to participants. REITs do not allow for the pass-through of losses. LO 11.b

Regarding the use of the term direct participation programs (DPPs) when referring to tax-sheltered investments, which of the following is not a DPP? A) A real estate limited partnership B) An oil and gas limited partnership C) An equipment leasing limited partnership D) A real estate investment trust (REIT)

A) I and III Explanation There are 100 basis points in each point. One point represents 1% of a bond's value; therefore, one basis point represents 0.01%, and 75 basis points would represent 0.75%. Because each point is worth $10, 75 basis points represents $7.50. LO 6.a

Seventy-five basis points are equal to which of the following? 0.75% 7.5% $7.50 $75.00 A) I and III B) II and III C) I and IV D) II and IV

D) I and IV Explanation Historic rehabilitation and government-assisted housing are two programs that offer potential tax credits. Tax credits are no longer available for equipment leasing, and while developmental oil and gas programs offer high intangible drilling costs, these are not investment tax credits. LO 11.e

Some limited partnership programs provide potential tax credits to partners. Which of the following typically provide potential tax credits? Rehabilitation of historic properties Equipment leasing Developmental oil and gas programs Government-assisted housing programs A) III and IV B) II and III C) I and II D) I and IV

B) an investment constraint Explanation Investment constraints are obstacles or restrictions that must be met in order to meet goals. In this case, we are dealing with a liquidity constraint—in seven months, cash will be necessary to make the purchase. LO 2.e

Tamika is a registered representative with Financial Engineers, LLC, a FINRA member broker-dealer. The firm uses an investment policy statement (IPS) to help design financial plans for their clients. One of Tamika's current clients plans to purchase a new boat seven months from now. When using the IPS, this would be considered A) a financial objective. B) an investment constraint. C) a long-term goal. D) an investment objective.

C) yield curve analysis. Explanation The plotting of bond yields results in a curve, usually one where the longer the time to maturity, the higher the yield. The term yield curve analysis is the proper way to describe comparing the yields of highly-rated corporate bonds to those of Treasury bonds. When the spread between the yields is narrow, economic conditions in the United States are generally favorable. If the spread (sometimes called the credit spread) widens, it is generally a sign of a worsening economy. ** This question deals with material not covered in your LEM, but it relates to recent rule changes and/or student feedback. LO 4.f

A bond analyst plots the yields of AAA corporate bonds and compares them to the yields of U.S. Treasury bonds with similar maturities. This is known as A) inverse yield analysis. B) yield comparison analysis. C) yield curve analysis. D) yield plot analysis.

D) makes a market in securities. Explanation A broker's broker acts as the agent in transactions by facilitating the movement of blocks of bonds. The broker's broker is allowed to conceal the identities of the contra-parties, thus protecting investment strategies. A broker's broker does not make a market in securities. LO 6.d

A broker's broker does all of the following except A) assists in placing securities. B) acts as agent for dealers. C) conceals the identity of the principals. D) makes a market in securities.

D) an income bond. Explanation Income (or adjustment) bonds carry the unique characteristic of requiring payment of interest only when the issuer's income is sufficient. They are used primarily for companies undergoing a financial restructuring, usually after a bankruptcy filing. Each of the other choices would require timely payment, and failure to do so could result in the company's failure. LO 5.a

A corporation coming out of a bankruptcy proceeding would probably find it most attractive to issue A) a subordinated debenture. B) a collateral trust certificate. C) a promissory note. D) an income bond.

A) 1 day Explanation Within 1 business day following receipt of the transfer instruction form (TIF), the carrying firm must validate the positions in the account and return the transfer instruction to the receiving member with an attachment showing all securities positions eligible for transfer through the Automated Customer Account Transfer Service (ACATS). LO 15.d

A carrying member—after receiving account transfer instructions from the receiving member—must validate the positions in the account within how many business days of receipt? A) 1 day B) 5 days C) 4 days D) 7 days

B) quantitative suitability. Explanation FINRA Rule 2111 places three obligations on members when determining if a specific recommendation to a customer is suitable. One of those obligations is quantitative suitability. Churning is generally defined as excessive trading in a customer's account. The registered representative, having control over a customer account, has to have a reasonable basis for believing that a series of recommended transactions, even if suitable when viewed in isolation, are not excessive and unsuitable for the customer when taken together. LO 2.f

A charge of churning would likely be brought against a registered representative who was found to have disregarded the FINRA rule on A) customer-specific suitability. B) quantitative suitability. C) reasonable-basis suitability. D) investment goal suitability.

A) overlapping debt Explanation The term overlapping debt refers to the issuer's proportionate share of the debt of other local governmental units that either overlap it (the issuer is located either wholly or partly with the geographical limits of the other units) or underlie it (the other units are located within the geographical limits of the issuer). In this case, the school district is probably within the geographical limits of the city. That's what coterminous means. LO 6.c

A city and school district are coterminous. When evaluating the debt issues of the city, the school district debt would be considered A) overlapping debt. B) a double-barreled bond. C) secondary debt. D) underlying debt.

C) should be refused. Explanation The order should be refused because it is impossible for it to be profitable. This is a bull call spread (but that is not the correct answer here because it has nothing to do with FINRA rules) and will become profitable when the spread widens. With strike prices of 100 and 105, it can never widen more than 5 points. If the client paid 5 points for the spread, once commissions are factored in, the client must lose money and certainly cannot profit. FINRA looks at this as an uneconomic position, and the firm should refuse to take the order. LO 10.h

A client with an options account contacts the registered representative handling the account with instructions to open the following spread: Buy 1 ABC 100 call and Sell 1 ABC 105 call at a 5-point debit. Under FINRA rules, this order A) will be executed at the next available trade meeting the 5-point limit. B) should be turned in immediately. C) should be refused. D) is for a bull call spread.

C) II and III Explanation Accelerated depreciation increases charged expenses during the early years of equipment life but decreases charged expenses during the later years. LO 13.c

A company's changing from straight line to accelerated depreciation will increase income in the early years. decrease income in the early years. increase income in the later years. decrease income in the later years. A) II and IV B) I and III C) II and III D) I and IV

C) $1,435,000 Explanation The corporation's $4 million operating income is taxed at a rate of 35%. For tax purposes, corporations can exclude 50% of all dividends received from domestic common and preferred stocks. Thus, 50% of the $200,000 received from preferred dividends is taxed at the 35% tax rate ($200,000 × 50% = $100,000). The $4 million in income plus the $100,000 in taxable dividends equals $4.1 million, and $4,100,000 multiplied by a 35% tax rate equals taxes of $1,435,000. LO 13.g

A corporation in the 35% tax bracket reports operating income of $4 million for the year. The firm also received $200,000 in preferred dividends from domestic corporations. Assuming no other items of income or expense, what is the company's tax liability? A) $1,756,000 B) $1,370,200 C) $1,435,000 D) $1,360,000

C) the issuer Explanation In a tender offer, the issuer is offering to buy back all or a portion of the issue at a stated price. The price of the tender is set by the issuer although the issuer may engage an underwriter to help it set the price. This could happen when interest rates have gone up, causing the price of the outstanding bonds to fall. From a practical standpoint, this would mean the corporation paying off debt at a price below face value. LO 4.b

A corporation plans to make a public tender for 50% of its outstanding bonds. The price of the tender will be set by A) the transfer agent. B) the trustee. C) the issuer. D) the paying agent.

C) $0 Explanation There is a lot more information in this question than necessary. Simply put, the deal went bankrupt—the asset was sold for less than the mortgage. That means the investor's $100,000 is totally lost. LO 11.f

A customer bought a 10% interest in a real estate limited partnership by investing $100,000. The partnership buys a $4 million property with the funds, making a down payment of $800,000 and financing the balance with a nonrecourse mortgage of $3.2 million. Subsequently, the partnership cannot meet the mortgage payment; the lender forecloses when the remaining mortgage balance is $3 million, auctioning off the property for $1 million. How much of the investment will the customer recover? A) $100,000 B) $10,000 C) $0 D) $32,000

A) a loss of $400 Explanation The 50 put expires because it is out of the money. The customer can close the position in the 65 put by purchasing it for its intrinsic value ($1,300) or, because the option is thirteen points in the money, it will be exercised. If exercised, the stock is purchased for $6,500 but is only worth $5,200. In either case, the investor loses $1,300. Because the account was credited $900 when the spread was established, there is a net $400 loss ($1,300 − $900). Please note that in test questions like this, the put writer who is exercised never buys and holds the stock; it is immediately sold at the current market price. Alternately, breakeven is 56 (65 − 9), and the spread is bullish. Therefore, the customer makes money above 56 and loses below 56. Because the stock is at 52 at expiration, the customer has a $400 loss (56 to 52). LO 10.e

A customer buys 1 XYZ Aug 50 put at 1 and sells 1 XYZ Aug 65 put at 10 when XYZ is at 58. If XYZ is at 52 at expiration, the customer has A) a loss of $400. B) a gain of $600. C) a gain of $400. D) a loss of $600.

C) $6,700 Explanation The client purchased 10 calls at 6.70 for a total of $6,700. Because—with the exception of LEAPS—options cannot be purchased on margin, the margin requirement is 100% of the premium. LO 10.g

A customer buys 10 Dec 91.50 calls on the Canadian dollar for 6.70. ($10,000 CD per contract). At the time of purchase, the spot rate for the Canadian dollar was 92.25. What is the margin requirement for the purchase? A) $3,350 B) $9,225 C) $6,700 D) $9,150

D) if LMN is below $41 at expiration. Explanation Breakeven is $41 ($45 − $4). The stock price must be above breakeven for the investor to make a profit. LO 10.h

A customer buys 300 LMN at $45 per share and writes 3 LMN Aug 45 calls at 4. The customer will profit under all of the following circumstances except A) if LMN is between $41 and $45 at expiration. B) if LMN remains at $45 through expiration. C) if LMN rises, and the calls are exercised. D) if LMN is below $41 at expiration.

B) Any gain in AC Growth Fund is taxable because the exchange is treated as a sale and a purchase Explanation The exchange is treated as a sale of the growth fund shares followed by a purchase of the income fund shares. The gain or loss is determined by comparing the cost basis of the growth fund shares with the net asset value at the time of exchange. Any difference is a capital gain or loss, even though the proceeds were immediately used to purchase the income fund. LO 8.e

A customer buys AC Growth Fund and enjoys a substantial paper capital gain. When he believes the market has reached its peak, he switches into AC Income Fund within the AC family of funds. He incurs a small service fee but is not charged an additional sales charge. What is the tax effect? A) Any gain or loss is deferred until he liquidates the AC Income Fund. B) Any gain in AC Growth Fund is taxable because the exchange is treated as a sale and a purchase. C) The tax basis of AC Income Fund is adjusted to reflect the gain in AC Growth Fund. D) It is a tax-free exchange.

D) two points of ordinary income and one point of capital gain. Explanation When a municipal bond is purchased in the secondary market at a discount, the annual accretion is taxed as ordinary income. The annual accretion is one point per year (four points divided by four years to maturity). Therefore, when the bond is sold two years later, its cost basis is 98. If the bond is sold at 99, there is a long-term capital gain of one point per bond. Also, there is ordinary income taxation on the accretion of two points. LO 6.f

A customer buys a municipal bond in the secondary market at 96 that has four years to maturity. Two years later, the customer sells the bond at 99. The tax consequences of this investment are A) two points of capital gain and one point of ordinary income. B) three points of ordinary income. C) three points of capital gain. D) two points of ordinary income and one point of capital gain.

C) a $50 capital gain Explanation The premium on the municipal bonds must be amortized. The tax rules require that when you purchase a bond at a premium, you have to reduce the cost basis of the bond each year. Even though there are five bonds in the question, here's the math on one bond and then we'll multiply by five to get the total amount. The investor buys the bond at 104 or $1,040 and the bond is due to mature in 20 years. Take the $40 premium divided by the 20 years to maturity and that will tell us the amount that we amortize/reduce the cost basis by each year. $40/20=$2. It then tells us that the bond is sold after 10 years. Ten years of amortization is $2 per year x 10 years = $20. That lowers the basis of the bond to $1,020 ( $1,040 - $20 = $1,020). The bonds are sold at 103 or $1,030, so the gain is $10 per bond times five bonds for a total gain of $50. LO 6.e

A customer buys five municipal bonds maturing in 20 years for 104. If he sells the bonds after 10 years at 103, the customer has A) a $50 capital loss. B) a $100 capital gain. C) a $50 capital gain. D) a $100 capital loss.

C) $35 Explanation This is a bull spread; the investor wants the stock to rise. Breakeven for put spreads is computed by subtracting the net premium (8) from the higher strike price (50). If it stays above the breakeven price of $42, they will profit. LO 10.e

A customer goes long an MMM Jan 40 put at 5 and writes an MMM Jan 50 put at 13. The customer will break even or profit when the market price is at all of the following except A) $48. B) $42. C) $35. D) $45.

B) I, II, III, and IV Explanation Any capital loss will offset a capital gain. LO 6.f

A customer has realized a capital gain from the sale of a municipal bond. To reduce her tax liability, the capital gain can be offset against a capital loss in which of the following investments? General obligations Equity securities Corporate bonds Collateralized mortgage obligation A) I and III B) I, II, III, and IV C) I and II D) II and III

A) Dividend treatment Explanation Of those listed, only dividend treatment can be identified as not being an advantage. While the expectation of receiving dividends is inherently good, dividends paid by REITs to their shareholders are not recognized as qualified and are therefore taxable to the investor at their full ordinary income tax rate. The shares are traded on exchanges or over the counter and are considered liquid, and having professionally managed assets should be a plus. While real estate valuation and price movements are subject to many forces, historically, real estate has provided some hedge against the movements of other equity securities. LO 11.b

A customer is considering adding a real estate investment trust (REIT) to her portfolio. She lists all of the following as advantages. You correct your customer and point out that one of them is not an advantage of investing in REITs. Which of the following is not an advantage of investing in REITs? A) Dividend treatment B) Using real estate as a potential hedge against the movement of other equity securities the customer owns C) Having a professionally managed portfolio of commercial real estate assets D) Being able to divest of the shares easily

A) $10,000 Explanation FINRA Rule 2310 limits compensation on the sale of a DPP to 10% of the offering price. That is the largest component of the offering expenses. Those are limited to 15% of the offering. DPPs are not covered by the 5% policy. Therefore, of the $100,000 invested, the business (the equipment leasing program) must receive at least $85,000 ($100,000 minus the maximum offering expenses of $15,000). If this question was about a DPP roll-up, then there is 2% maximum to the member if recommending the client vote in favor of the roll-up. LO 11.h

A customer of a member firm has just invested $100,000 into an equipment leasing DPP. Under FINRA rules, the maximum compensation allowable to the firm is A) $10,000. B) $5,000. C) $15,000. D) $2,000.

A) be changed for individual transactions, or going forward, for all transactions Explanation Once payment and delivery instructions are established at the time the account is opened, they can be changed for any individual transaction or for all transactions going forward. LO 1.a

A customer opens an account, and payment and delivery instructions are established. Beyond the opening of the account, these instructions may A) be changed for individual transactions, or going forward, for all transactions. B) be changed at any time for all transactions going forward. C) not be changed unless a new account is established. D) be changed for individual transactions only.

D) Maximum gain is $300; maximum loss is $200; breakeven point is $42. Explanation The first step is to identify the position. This is a credit put spread. It is a credit spread because the option sold brought in a higher premium than the one purchased. The credit of $300 is the most the investor can make. This is a bullish spread (the customer bought the low strike price and sold the high strike price). If the customer is correct and the stock rises above $45, the options will expire unexercised and the customer will keep that net credit of $300. If the customer is wrong and the price of the ALE stock falls below $40, the short put at 45 will be exercised, causing the customer to purchase the stock at $45. Then, the customer will exercise the long 40 put and sell that stock at $40. This results in a loss of $500 reduced by the $300 net credit, or $200 maximum loss. It is always easier to recognize that the maximum loss is the difference in strike prices minus the maximum profit. In this question, the spread is 5 points and the maximum profit is the credit of 3 points. That makes the maximum loss the remaining 2 points. Breakeven follows the put-down rule. Subtract the net premium from the higher strike price ($45 - 3 = $42). LO 10.h

A customer opens the following options position: Long 1 ALE Feb 40 put @3¼; short 1 ALE Feb 45 put @6¼. What is the customer's maximum gain, maximum loss, and breakeven point? A) Maximum gain is $200; maximum loss is $300; breakeven point is $43. B) Maximum gain is $200; maximum loss is $300; breakeven point is $42. C) Maximum gain is $300; maximum loss is $200; breakeven point is $43. D) Maximum gain is $300; maximum loss is $200; breakeven point is $42.

B) Maximum gain is $200; maximum loss is $3,800, breakeven point is $38. Explanation The first step is to identify the position. This long stock with a short call (i.e., a covered call position). Breakeven is the customer's net cost. The price of the stock ($40) minus the premium ($2) received equals the $38 per share breakeven point. The strategy is to generate some income with a little protection against a decline in the price of the CDL stock. The premium income is the most this client can make. If the stock should rise well above the cost of $40 per share, the short call will be exercised and the customer will deliver the stock purchased at $40 and receive $40. Regardless of how high the stock price rises, this customer can never make more than the $2 premium. If the stock's price should decline, the call will expire unexercised. That 2-point premium protects the long stock, but only for those 2 points. Once the market price falls below $38 (the breakeven point), it is all a loss for the customer, down to a maximum $3,800 if the price drops to zero. Why doesn't the breakeven follow the "call-up" rule? That rule applies when the only positions are options. Once there is a long or short stock position along with an option position, it is the stock controlling the breakeven. LO 10.h

A customer opens the following positions: Buy 100 shares of CDL @$40; sell 1 CDL Apr 40 call @2. What is the customer's maximum gain, maximum loss, and breakeven point? A) Maximum gain is $200, maximum loss is unlimited; breakeven point is $38. B) Maximum gain is $200; maximum loss is $3,800, breakeven point is $38. C) Maximum gain is $200; maximum loss is $3,800; breakeven point is $42. D) Maximum gain is unlimited; maximum loss is $3,800; breakeven point is $38.

D) Maximum gain is unlimited; maximum loss is $400; breakeven point is $64. Explanation The first step is to identify the position. This is a long stock position with a protective put. That is, the customer has purchased the stock and purchased a put to protect the downside. Using an option as a form of insurance is the primary reason why the industry refers to the price of an option as the premium. On questions with stock and an option, it is usually best to compute the breakeven point first. Breakeven is when the long stock can be sold at the customer's total cost. That cost is the price of the stock ($60) plus the price paid for the option ($4), or $64. If the stock should rise above $64, the customer will let the 60 put expire and maintain the long stock position. An investor with a long stock position has unlimited potential gain. If the stock price should decline, no matter how low it drops, the customer can exercise the long put and sell the stock for $60 per share. That means the maximum loss is the premium paid for the option, $400. Why doesn't the breakeven follow the "put-down" rule? That rule applies when the only positions are options. Once there is a long or short stock position along with an option position, it is the stock controlling the breakeven. LO 10.h

A customer opens the following positions: Buy 100 shares of HDH @60; buy 1 HDH Feb 60 put @4. What is the customer's maximum gain, maximum loss, and breakeven point? A) Maximum gain is unlimited; maximum loss is $5,600; breakeven point is $56. B) Maximum gain is $6,400; maximum loss is $400; breakeven point is $64. C) Maximum gain is $400; maximum loss is $5,600; breakeven point is $56. D) Maximum gain is unlimited; maximum loss is $400; breakeven point is $64.

D) Tender the bonds to the corporation Explanation First of all, recognize that the investor purchased 10 of the debentures. They have a coupon of 9% and mature in 2045. None of that is relevant to answering the question, but we want to be sure you understand the terminology.The option most beneficial to the investor is tendering the debentures to the corporation for $10,100 (10 times $1,010). If the debentures were sold on the market, the investor would receive $9,800 (10 times $980). If the debentures were converted into common, the investor would receive 400 common shares (40 shares per debenture times 10) that could be sold for their current price of $24, for a total of $9,600. LO 5.d

A customer purchased 10 ABC 9s of 2045 convertible debentures at 99. The debentures are callable at 101. The conversion ratio is 40. Some time later, the debentures are called while the common is trading at $24 and the debenture is trading at 98. Which of the following options would be most beneficial to the customer? A) Sell the bonds B) Wait for a better offer from the corporation C) Convert the bonds and sell the common stock D) Tender the bonds to the corporation

B) $500 Explanation The customer buys stock at $17.50 and sells his shares at $19 for a gain of $300. In addition, the customer keeps the $200 in premiums, for an overall gain of $500. LO 10.h

A customer purchases 200 shares of XYZ at 17.50 and writes 2 XYZ Jan 20 calls at 1. At expiration, with the stock trading at 19, the options expire worthless. If the customer sells his long stock at the current market price, the gain is A) $250. B) $500. C) $350. D) $700.

C) sell an FLB Mar 35 put Explanation Straddles involve options of different types, but both options must be of the same series. An option series has the same strike price, expiration date, and underlying security. LO 10.f

A customer sells an FLB Mar 35 call. To establish a straddle, she would A) sell an FLB Mar 40 call. B) buy an FLB Mar 35 put. C) sell an FLB Mar 35 put. D) buy an FLB Mar 40 call.

D) Buy 10 ZOO calls Explanation To protect the profit on the short stock position, the customer must be able to buy stock at the existing low price if the market moves up. By purchasing calls (say, at a $25 strike price), the customer can capture existing profit by exercising and buying stock at $25, regardless of how high the market moves. LO 10.d

A customer sells short 1,000 ZOO at $30 per share. If the ZOO stock declines to $25 per share, and the customer is worried the stock may reverse its trend, what should the customer do? A) Write 10 ZOO puts B) Write 10 ZOO calls C) Buy 10 ZOO puts D) Buy 10 ZOO calls

D) a principal transaction. Explanation This is an example of a riskless principal trade. The member firm that bought the shares already knows who it will sell to and did not incur inventory risk. While this may seem like an agency trade, the firm is still selling from its own inventory, making it a principal transaction. LO 13.g

A customer wishes to purchase 100 shares of ABC for $35 but does not wish to be charged a commission. To accommodate this request, the member firm purchases the shares for the customer in its proprietary trading account and sells the shares to the customer with a markup. This trade is A) an agency transaction. B) a cross transaction. C) a proceeds transaction. D) a principal transaction.

B) any 12b-1 charge does not exceed .25%. Explanation Mutual fund can call themselves no-load as long as they do not have a 12b-1 charge that exceeds .25%. In addition, there cannot be any front-end load. LO 8.d

A mutual fund can use the term, "no-load" as long as A) there is no 12b-1 charge. B) any 12b-1 charge does not exceed .25%. C) any 12b-1 charge does not exceed 1.00%. D) any 12b-1 charge does not exceed .75%.

B) 60% U.S. government bond funds, 30% broad market index funds, 10% growth funds Explanation Moving toward retirement, the reallocation should move the portfolio away from equities and sector funds toward fixed-income funds. U.S. government securities funds accommodate that, and the U.S. government securities within the fund are considered safe with no default risk. Coupling the U.S. government bond funds with smaller percentages in broad market index and growth funds that mirror the market can help the portfolio keep pace with inflation. Remember that utilizing municipal securities in a tax-favored account, such as an IRA, would be considered unsuitable because the interest paid by municipal bonds is already tax free. LO 14.a

A customer, age 62, wants to retire at age 64 and has accumulated investments in an IRA currently valued at $500,000. The IRA portfolio consisting of all mutual funds is allocated as follows: 70% growth funds, 10% corporate bond funds, and 20% sector funds. Still wanting to use mutual funds, which might be the most suitable reallocation of the portfolio as this customer nears retirement? A) 70% municipal bond funds, 20% broad market index funds, 10% sector funds B) 60% U.S. government bond funds, 30% broad market index funds, 10% growth funds C) 30% municipal bond funds, 30% corporate bond funds, 40% growth funds D) 80% broad market index funds, 10% corporate bond funds, 10% U.S. government bond funds

C) $1.25. Explanation Treasury notes and bonds are quoted in fractions of 32nds. The spread between the bid and the ask is 4/32nds. In simpler terms, that is 1/8th. Each point is $10.00, so this 1/8th of $10.00 is equal to $1.25. LO 7.b

A dealer in U.S. government securities quotes a 5-year Treasury note at 89.12-89.16. In dollars, that represents a spread of A) $0.04. B). $4.00. C) $1.25. D) $0.125.

D) Writing uncovered calls Explanation The prudent investor rule permits a fiduciary to invest in securities that a reasonably prudent investor, seeking income and preservation of capital, might invest in for her own account. Writing uncovered (naked) calls that have an unlimited loss potential would not meet the standard of the rule. LO 10.d

A fiduciary acting under the prudent investor rule should recognize that one of the following transactions would not be acceptable or meet the standards of the rule. Which is it? A) Purchasing BBB-rated debentures B) Purchasing shares of a tech sector mutual fund C) Buying sector funds D) Writing uncovered calls

B) I and III Explanation When a bond is referred to by a yield percentage, it is the coupon (nominal or stated) yield being referenced. Basis yield refers to yield to maturity (YTM). Hence, a 6% bond currently trading with a 6.5% YTM is correct. LO 4.e

A registered representative mentions a particular 6% municipal bond quoted on a 6.5% basis. Which of the following is correct? Six percent is the bond's coupon. Six percent is the bond's current yield. Six-and-a-half percent% is the bond's yield to maturity. Six-and-a-half percent% is the bond's current yield. A) I and IV B) I and III C) II and III D) II and IV

A) borrows money from the business Explanation The general partner manages the business and acts as agent for the business. The general partner may loan money to the partnership at a reasonable rate of interest but may not borrow from the partnership. LO 11.d

A general partner is considered to have a conflict of interest with the business of a limited partnership if she A) borrows money from the business. B) manages the business. C) loans money to the business. D) acts as agent for the business.

A) borrows money from the business Explanation The general partner manages the business and acts as agent for the business. The general partner may loan money to the partnership at a reasonable rate of interest but may not borrow from the partnership. LO 11.d

A general partner is considered to have a conflict of interest with the business of a limited partnership if she A) borrows money from the business. B) manages the business. C) loans money to the business. D) acts as agent for the business.

A) Municipal bond ETF Explanation Municipal bonds and GNMAs are the two conservative choices offered here. However, distributions of interest from the GNMA fund will be taxed at the federal, state, and local level. Income distributions from the municipal bond ETF will be tax exempt at the federal level, reducing tax exposure at their income level, and is the better choice. Direct participation programs are not conservative or liquid, and special situation mutual funds offer no reduced tax exposure. LO 8.g

A high net worth couple in their 40s earning a combined $480,000 annually have $100,000 they would like to conservatively invest. Given their income bracket, they stress reducing tax exposure and also note that having access to the funds is important. Which of the following would be the most suitable investment? A) Municipal bond ETF B) Direct participation program C) GNMA fund D) Special situation mutual fund

D) bearish. Explanation When a stock is rising in price toward its resistance level, generally sellers will enter the market and prevent it from breaking through. This would therefore be a bearish indicator—that the stock has reached the highest price it will go. While breakouts can occur, they are considered significant events. LO 13.e

A registered representative noticed that a stock was approaching its resistance line. This is generally considered A) bullish. B) ambiguous. C) neutral. D) bearish.

A) the grey market Explanation Securities trading on the "grey market" are not quoted on any U.S. quotation system. Broker-dealers are not willing or able to publicly quote these securities because of a lack of investor interest, company information availability, or regulatory compliance. Without quotes and reports to the SEC, there is little if any transparency. **This question deals with material not covered in your LEM, but it relates to recent rule changes and/or student feedback. LO 3.h

A key term in the SEC's lexicon is transparency. It is often said that "disclosure is the religion of the SEC." When investigating a security for a potential recommendation to a customer, it is likely that the least transparency exists when the stock trades in A) the grey market. B) the OTC market. C) the listed market. D) the secondary market.

D) a prime brokerage account. Explanation A prime brokerage account is one in which a customer—generally an institution—selects one member firm (the prime broker) to provide custody, trading, and other services, while other firms, called executing brokers, typically execute most of the trades placed by the customer. LO 1.a

A large institution dealing with several different broker-dealers would probably find it beneficial to open A) a combined cash and margin account. B) an institutional brokerage account. C) an accredited investor account. D) a prime brokerage account.

A) Treasury notes, municipal bonds, GNMAs Explanation In light of their low risk tolerance, U.S. government securities would certainly be suitable, and the time frame noted for retirement allows for middle term T-notes to be useful. Given their higher income level, tax-free municipal bonds could also have a place in the portfolio. Longer term GNMAs would accommodate monthly income, should that be desirable upon retirement. The remaining product suggestions are either illiquid (DPPs) or do not align with their risk aversion (common, preferred, options, and REITs). LO 11.h

A married couple both hoping to retire within the next five to seven years have expressed having a low-risk tolerance regarding the stock market. They have a combined income of $350,000. Given this information, which of the following portfolio mixes would be most suitable? A) Treasury notes, municipal bonds, GNMAs B) Direct participation programs, real estate investment trusts, preferred stock C) Treasury bills, corporate bonds, preferred stock D) Treasury bills, common stock, options

B) A treasury STRIP scheduled to mature in 16 years Explanation Treasury STRIPs are zero-coupon bonds, backed in full by the U.S. government. Purchased at a discount and maturing at face value in the future, they are suitable investments for those wishing to save for anticipated future expenses, such as college tuition. A CMO maturing in five years doesn't align with the time horizon for this child's college education and carries other unsuitable risks. A money market fund would hardly meet the growth requirement needed to meet college tuition needs. For exam purposes, municipal bonds are a suitable choice only when something in the question indicates that the investor is in a high income tax bracket. LO 12.d

A married couple with a two-year-old child have $25,000 to deposit towards an investment to help meet the financial obligations for the child's college education. Given the following choices, which of the following is likely the most suitable investment? A) A diversified portfolio of insured municipal bonds with an average duration of 18 years B) A treasury STRIP scheduled to mature in 16 years C) A collateralized mortgage obligation (CMO) tranche rated AAA and scheduled to mature in five years D) A money market mutual fund

A) a capital gain of $5 per bond Explanation Municipal bonds bought at a premium, either in the new issue or secondary market, must be amortized. The amount of the premium is 2½ points, or $25. As the bond has five years to maturity, the annual amortization amount is $5 per bond. After two years, the bond's basis has been amortized down to 101½. At that point, a sale at 102 generates a capital gain of $5 per bond. LO 6.e

A municipal bond is purchased in the secondary market at 102½. The bond has five years to maturity. Two years later, the bond is sold for 102. The tax consequence to the investor is A) a capital gain of $5 per bond. B) a capital loss of $20 per bond. C) no capital gain or loss. D) a capital loss of $5 per bond.

C) a double-barreled bond Explanation When a municipal bond is backed by both a source of revenue and the taxing ability of the issuer, this is referred to as a double-barreled bond. LO 6.b

A municipal bond, issued with a covenant that states, "If revenue collections are not sufficient to meet debt service requirements, the issue will be backed by the full faith and credit of the municipality," is known as A) a Section 8 bond. B) a contingent liability bond. C) a double-barreled bond. D) a moral obligation bond.

D) an associate of a broker-dealer engaged in municipal securities representative activities other than retail sales. Explanation An MFP is an associate of a broker-dealer engaged in municipal securities representative activities other than retail sales. Those activities can include the solicitation of municipal bond business. MFPs are subject to the MSRB reporting rules regarding gifts to elected officials and political parties (MSRB Rule G-37). LO 6.h

A municipal finance professional (MFP) is A) an elected official of a municipality with decision-making responsibilities regarding municipal issues. B) an employee of the Municipal Securities Rulemaking Board (MSRB) responsible for broker-dealer compliance regarding MSRB rules. C) a registered representative engaged in the sale of municipal securities to public customers. D) an associate of a broker-dealer engaged in municipal securities representative activities other than retail sales.

C) an employee of a broker-dealer engaged in municipal security representative activities other than retail sales or who solicits municipal securities business for the broker-dealer. Explanation Per the MSRB, an MFP is an associated person of a broker-dealer who is primarily engaged in municipal securities representative activities other than retail sales to individuals, who solicits municipal securities business for the broker-dealer, or who is in the supervisory chain above MFPs. LO 6.h

A municipal finance professional (MFP) is A) an employee of the Municipal Securities Rulemaking Board (MSRB) specializing in seeing that broker-dealers adhere to the MSRB rules and regulations regarding the sales of municipal bonds to retail customers. B) an elected official of a municipality having some decision-making authority regarding new municipal bond issues. C) an employee of a broker-dealer engaged in municipal security representative activities other than retail sales or who solicits municipal securities business for the broker-dealer. D) employed by a municipality to oversee the issuance of municipal bond

B) retain high salaried key employees. Explanation Key employees in higher tax brackets do not get the full benefit of bonuses or pay increases because so much goes to pay the taxes. The attractiveness of the deferred compensation program is that those funds are deferred until a later time, such as retirement, when it is expected that the individual will be in a lower tax bracket. Nonqualified plans are not covered by ERISA. This enables the company to "pick and choose." The deferred compensation plan doesn't affect current corporate taxation because nothing is expended; the deduction does not come until the compensation is paid in the future. Deferred compensation plans are most often used to supplement, not replace, qualified plans. LO 1.f

A nonqualified deferred compensation plan is often used by corporations to A) Replace an outdated qualified plan. B) retain high salaried key employees. C) reduce their current income tax burden. D) comply with ERISA.

A) A hedge fund Explanation One of the distinct characteristics of a hedge fund is the fee structure. A typical offering will describe the fee as "2 & 20." This means that the base fee is 2% of assets, with a bonus of 20% of the profits once the return reaches a specific range. It is unlikely that you will see this feature on your exam anywhere other than with hedge funds. It creates a strong incentive for the fund's manager to implement strategies that may produce high returns but carries with it a higher risk for the investors. There are no management fees with UITs and mutual fund fees are almost always below 1% (most significantly less that that). LO 12.b

A performance-based management fee is normally associated with which of the following pooled investments? A) A hedge fund B) An asset allocation fund C) A unit investment trust D) A balanced fund

C) a fiduciary Explanation Fiduciary is the term that describes the legal position of trustees, custodians, and most investment advisers. This is a case where you choose the most complete response. LO 2.g

A person legally responsible for the handling of the financial assets of another, such as an executor or guardian, is usually called A) a custodian. B) a trustee. C) a fiduciary. D) an investment adviser.

A) taking a long position in euro calls Explanation There are no listed options in the U.S. dollar. That reduces your choice to a long euro put or call. Because the contract will be paid for in dollars, the producer is concerned that the dollar will fall against the euro. Or, stated another way, the concern is that the euro will rise against the dollar so that the $10 million will not buy as many euros as on the day the contract was signed. When one is afraid the price of asset will rise, such as those who take a short position in a stock, the best protection is buying a call. The easiest way to remember this is through the acronym, IPEC - Importers buy Puts and Exporters buy Calls. This is used when the party involved in the question is in a foreign country. Because we are dealing with a French exporter, buying calls on the local currency offers the best protection. LO 10.g

A producer of fine French wines has just signed a contract to export $10 million of wine to a distributor in the United States. Using listed foreign currency options, this producer would have the best protection against currency risk by A) taking a long position in euro calls. B) taking a long position in U.S. dollar calls. C) taking a long position in U.S. dollar puts. D) taking a long position in euro puts.

D) $200,000. Explanation Losses may only be claimed to the extent of tax basis. The initial $800,000 was divided 10 ways, so each LP had a basis of $80,000. To this was added the share of the financing of $1.2 million. That is another $120,000 basis (10% of $1.2milion) bringing the total to $200,000 ($80,000 + $120,000). That is the maximum loss that can be claimed. It is important to note that nonrecourse financing adds to basis only in RELPs. Because the loan adds to the basis of all LPs equally, you could also solve this by taking the total $2 million investment and dividing it by 10 to arrive at the same $200,000. LO 11.f

A real estate limited partnership is created for $800,000 with 1 general partner and 10 limited partners. Each of the limited partners has an equal 10% share. The proceeds are used to purchase an office building for $2 million. The additional financing is provided by a nonrecourse bank loan. Economic conditions cause the occupancy rate to fall dramatically, and the partnership is dissolved as insolvent. Each limited partner may claim a loss of A) $80,000. B) $120,000. C) $2,000,000. D) $200,000.

C) II and IV Explanation Contributions to a Coverdell ESA are made with after-tax dollars. Distributions used for qualified educational expenses are tax free. LO 1.g

A registered representative is explaining the characteristics of a Coverdell Education Savings Account (ESA) to a customer. Which of the following statements regarding this type of savings account is correct? Contributions are tax deductible. Contributions are not tax deductible. When used for qualified educational expenses, withdrawals are taxable. When used for qualified educational expenses, withdrawals are not taxable. A) I and III B) I and IV C) II and IV D) II and III

B) I and IV Explanation Corporate accounts are generally those established by the officers of a corporation. Such accounts require a copy of the corporate resolution naming the authorized person(s) and account trading limits (if any). If it is to be a margin account, a copy of the corporate charter and a signed margin agreement are also required. LO 2.g

A registered representative is opening both cash and margin accounts for a corporation. Which of the following documents will he need? The corporation's charter and bylaws A copy of the corporation's most recent balance sheet The corporation's last three profit and loss statements The name(s) of natural persons authorized to trade the account A) II and III B) I and IV C) II and IV D) I and III

C) The registration fees for the seminar Explanation Under the safe harbor provisions of Section 28(e) of the Securities Exchange Act of 1934, broker-dealers are permitted to extend seminar invitations to investment advisers with whom they do or hope to do business. The only expense reimbursable by the broker-dealer is the fee to attend the seminar. LO 14.d

A registered representative of a FINRA member firm has developed a LinkedIn friendship with a registered investment adviser. This has resulted in the investment adviser directing transactions for many of their clients to this representative's broker-dealer. The broker-dealer is promoting an all-day seminar with presentations to be delivered by a number of outstanding economists and securities analysts. The seminar location is in a hotel ballroom down the street from the member firm's office. The firm has invited the investment adviser to attend as its guest. That location requires the adviser to fly in the night before and stay at the hotel. As the broker-dealer's guest, which of the following expenses are reimbursable by the broker-dealer without violating the safe harbor provisions of Section 28(e)? A) The registration fees for the seminar plus all of the travel expenses B) The travel expenses, but not the registration fee C) The registration fees for the seminar D) The registration fees for the seminar plus the hotel room for the night

B) These funds, called funds of hedge funds, eliminate all of the risks associated with hedge funds Explanation Because the portfolio of the registered fund consists of shares of nonregistered hedge funds, virtually all of the risks associated with hedge funds are transferred to the mutual fund. Funds of hedge funds allow all investors—not just accredited investors—to have access to hedge fund investments, and they are likely to have lower initial investment requirements, making that access even easier. To divest of fund of hedge fund shares, the issuer would have to redeem them from the investor, as is the case with all registered mutual funds. LO 12.b

A registered representative speaking to a customer is explaining registered funds that invest in nonregistered hedge funds. Which of the following statements is not correct? A) To divest of your fund of hedge fund investment, the shares will need to be redeemed by the mutual fund issuer. B) These funds, called funds of hedge funds, eliminate all of the risks associated with hedge funds. C) Hedge funds are directly available to sophisticated (accredited) investors, while funds of hedge funds allow all investors to invest in hedge funds indirectly. D) These funds generally allow purchases with an initial investment that is lower than what is required to invest directly in a hedge fund.

D) a functional allocation. Explanation Functional allocation is a sharing arrangement in which the general partner pays for all tangible drilling costs (capitalized costs), and the limited partners pay for all intangible drilling costs (deductible costs). LO 11.f

A sharing arrangement in which only deductible costs are apportioned to the investor, with the sponsor bearing all capitalized costs is called A) an overriding royalty arrangement. B) a reversionary sharing arrangement. C) a carried interest. D) a functional allocation.

B) The purchase of ADRs representing the shares exposes me to currency risk Explanation From a suitability perspective, correcting any inaccuracies about an investment that an investor might have is important. Currency risk cannot be avoided when investing in foreign companies, either directly or using ADRs. While ADRs trade on U.S. exchanges, foreign shares do not, and ADR issuers generally do not pass on voting rights to the ADR holders. LO 3.g

A sophisticated investor wants to purchase stock of a foreign company or an American depositary receipt (ADR) representing the shares of that company. The purchase would align with the investor's goal of growth and income, but he makes several statements about the potential purchase, and only one of them is accurate. You feel it is important to point out and discuss from a suitability perspective which statements were and were not accurate. Which of the following is the accurate statement? A) I can trade the foreign shares right here on U.S. exchanges. B) The purchase of ADRs representing the shares exposes me to currency risk. C) The direct purchase of the foreign stock shares eliminates currency risk. D) With the ADRs, I'll have voting rights just like I would if I purchased the shares directly.

B) structured products Explanation In Notice to Members 05-59, FINRA defined a structured product as "securities derived from or based on a single security, a basket of securities, an index, a commodity, a debt issuance and/or a foreign currency." The most important thing for you to know for the exam is that these generally carry higher risk than other debt securities. These should be recommended only when the registered representative has a thorough understanding of the product and believes it is suitable for the specific investor. Yes, these are high-risk investments, but that is not the term used to describe them. LO 4.g

A term used to define certain alternative forms of debt financing, such as equity-linked notes (ELNs) and exchange-traded notes (ETNs), is A) combination products. B) structured products. C) high-risk investments. D) principal protected products.

A) marketability risk. Explanation It is difficult to sell a large block of securities in a thinly traded stock without a substantial discount to market price. This is known as liquidity or marketability risk. Technically, the terms are not identical, but for test purposes, consider it so. LO 14.a

A wealthy client owns a large percentage of a thinly traded common stock. When this client wants to sell a major portion of her securities, she will immediately face A) marketability risk. B) market risk. C) credit risk. D) interest rate risk.

A) To profit in this situation, the investor should buy the bonds and short the stock. Explanation With a conversion price of $40, the bond is convertible into 25 shares of ABC common stock ($1,000 / $40 = 25 shares). As the common stock is currently trading at $45 per share, the value of the stock as converted would be $1,125 (25 shares × $45 = $1,125), which is greater than the current price of the bond ($1,020). Therefore, the bond and the stock are not at parity. An investor could profit in this situation by shorting the stock and buying an equivalent number of bonds. A bond could be purchased for $1,020 and immediately converted into stock worth $1,125—a risk-free profit opportunity. LO 5.d

ABC Corporation has outstanding a 7.75% convertible debenture currently trading at 102. The bond is convertible into common stock at $40. ABC stock is trading $45 per share. Which of the following statements is true? A) To profit in this situation, the investor should buy the bonds and short the stock. B) The bond is at parity with the stock. C) To profit in this situation, the investor should buy the stock and short the bonds. D) An arbitrage opportunity does not exist in this situation.

D) A security with negative correlation Explanation MPT states that the lower an asset's correlation to the overall portfolio, the greater the diversification. Greater diversification equals lower risk. Adding a security with a negative correlation offers the lowest risk. Adding a security to the portfolio that has a -1.0 correlation reduces the overall risk due to the greater diversification. The higher the security's positive correlation, the less diversification, leading to greater risk. LO 13.b

According to modern portfolio theory (MPT), an investor can reduce the level of risk through diversification. Adding which of the following securities would tend to reduce the portfolio's overall risk? A) A security with positive correlation B) A security with a leveraged correlation C) A security with equal correlation D) A security with negative correlation

A) 20% Explanation The leverage is the extent to which borrowed funds make up the company's total capital. Total capital is the value of the equity and debt financing combined. Acme has issued $50 million of debentures (debt capital) and $200 million of equity capital (the common stock). That makes the total capitalization of Acme equal to $250 million. The leverage is $50 million divided by $250 million, or 20%. An analyst would consider this conservative leverage. LO 13.d

Acme Pharmaceuticals previously had issued $200 million of common stock in an IPO. A year later, it issued $50 million of debentures at par value. Acme's leverage is what percentage of its total capital? A) 20% B) 50% C) 25% D) 400%

D) inventory. Explanation Inventory is considered a current asset—not a fixed asset—because the company expects to convert its inventory into cash within a short time. The other choices are fixed assets and cannot be liquidated easily. LO 13.c

All of the following appear on a corporation's balance sheet as fixed assets except A) computer equipment. B) furniture. C) real estate. D) inventory.

B) hedging a long stock position against falling prices Explanation Long stock positions are best hedged with the purchase of a long put. Call buyers protect short stock positions, speculate on the upward movement of a stock's price, diversify their holdings, and delay the decision to buy stock because of the expiration period. LO 10.d

All of the following are call buyers' objectives except A) diversifying holdings. B) hedging a long stock position against falling prices. C) delaying a decision to buy stock. D) speculating for profit on the rise in price of stock.

A) depletion. Explanation A depletion allowance makes up for the using up of a natural resource. Real estate limited partnerships do not have depletion allowances. Both real estate and oil and gas partnerships offer limited liability, depreciation allowances, and deferred receipt of income and capital gains. LO 11.f

All of the following are characteristics of both oil and gas, as well as real estate limited partnerships, except A) depletion. B) limited liability. C) depreciation. D) deferral of benefits.

C) growth of capital Explanation Growth of capital is an investment objective. The other choices represent obstacles (constraints) that might keep the investor from fulfilling that objective. LO 2.e

All of the following are investment constraints except A) investor preferences. B) liquidity. C) growth of capital. D) time horizon.

C) facilitate trading foreign securities in U.S. markets by U.S. citizens living in the United States Explanation ADRs make trading in foreign securities easier in U.S. markets for U.S. investors. LO 3.g

An American depositary receipt (ADR) is used to A) finance foreign trade in which U.S. citizens are engaged. B) facilitate trading U.S. securities in foreign markets by U.S. citizens living abroad. C) facilitate trading foreign securities in U.S. markets by U.S. citizens living in the United States. D) sweeten a bond offering.

D) the market for mortgage-backed securities is illiquid. Explanation As publicly traded securities, liquidity risk is not a major concern to investors in mortgage-backed securities. When interest rates decline, there is generally an increase in mortgage refinancing and that results in investors receiving repayment of principal ahead of schedule. That is prepayment risk. Although it is nice to get the money back, there is reinvestment risk because these lower interest rates mean that reinvesting the principal into new securities will now be at a lower return than previously earned. The opposite happens when interest rates increase. Homeowners are unlikely to refinance, causing principal repayments to slow. Without the repayment, investors holding these securities are receiving below market returns. This is an example of extension risk. As is the case with any loan, there is always default risk. Recent history has shown that it is possible for a large number of foreclosures where the lender does not recoup the full principal. LO 12.c

All of the following are risks of investing in publicly traded mortgage-backed securities except A) borrowers might default on their mortgage payments. B) rising interest rates might extend the date of repayment of principal. C) falling interest rates might accelerate early repayment of principal. D) the market for mortgage-backed securities is illiquid.

D) they must pass along losses to shareholders. Explanation REITs engage in real estate activities and can qualify for favorable tax treatment if they pass through at least 90% of their net investment income to their shareholders. While they can pass through income, they cannot pass through any losses; they are not direct participation programs. LO 11.b

All of the following are true of real estate investment trusts (REITs) except A) they must, to qualify under Subchapter M, distribute at least 90% of their net investment income. B) they must invest at least 75% of their assets in real estate-related activities. C) shares are publicly traded. D) they must pass along losses to shareholders.

D) real estate taxes. Explanation Special tax bonds are sometimes included in the larger and more general category of revenue bonds. Bonds supported from the proceeds of specified income generators, such as gasoline, cigarettes, liquor, and business licenses, are special tax bonds. Ad valorem (real estate) taxes never service special tax bonds. LO 6.b

All of the following may be used to service special tax bond issues except A) excise taxes. B) gasoline taxes. C) business license taxes. D) real estate taxes.

D) real estate taxes. Explanation Special tax bonds are sometimes included in the larger and more general category of revenue bonds. Bonds supported from the proceeds of specified income generators, such as gasoline, cigarettes, liquor, and business licenses, are special tax bonds. Ad valorem (real estate) taxes never service special tax bonds. LO 6.b

All of the following may be used to service special tax bond issues except A) excise taxes. B) gasoline taxes. C) business license taxes. D) real estate taxes.

C) short uncovered calls. Explanation A short uncovered call has unlimited potential loss because, theoretically, a stock's price can rise indefinitely. The potential loss on a short uncovered put can be large but is limited to the put's strike price minus the premium received per share. The potential loss on a long call or a long put is limited to the premium paid. LO 10.d

All of the following option positions have limited potential loss except A) Long calls. B) short uncovered puts. C) short uncovered calls. D) long puts.

C) FOCUS reports. Explanation FOCUS reports (it will not be tested, but the acronym stands for Financial and Operational Combined Uniform Single) are part of the large group of records with a three-year retention requirement. Blotters, ledgers, and most records containing detailed information about customers have a six-year requirement. LO 15.e

All of the following records must be kept for six years except A) the general ledger. B) customer account records. C) FOCUS reports. D) trade blotters.

A) variable annuities will protect an investor against capital loss. Explanation As the name implies, the investment performance of a variable annuity's portfolio (separate account) can vary, and the investor bears the risk of any potential decline in its value. Many variable annuities invest the separate account in mutual funds. LO 9.a

All of the following statements concerning a variable annuity are correct except A) variable annuities will protect an investor against capital loss. B) the invested money will be professionally managed according to the issuer's investment objectives. C) a majority vote from the shareholders is required to change the investment objectives. D) an insurance and securities license is needed to sell a variable annuity.

D) holders receive a higher interest rate. Explanation Because of the possibility of participating in the growth of the common stock through an increase in the market price of the common, the convertible can be issued with a lower interest rate. The interest rate on a convertible, just as with any other fixed-income security, does not change.

All of the following statements regarding convertible bonds are true except A) the issuer pays a lower interest rate. B) holders receive a fixed interest rate. C) holders may share in the growth of the common stock. D) holders receive a higher interest rate.

B) an increase in municipal assessed valuations. Explanation An increase in assessed property values would theoretically mean an increase in taxes collected, thus increasing a municipality's credit standing. LO 6.b

All of the following would be indications of a deteriorating credit situation except A) an increase in tax delinquencies. B) an increase in municipal assessed valuations. C) an increase in personal bankruptcies. D) an increase in the per capita debt.

D) a warehouse. Explanation Current assets are those that are either cash or expected to generate cash within the next year. Warehouses are fixed assets used for many years. LO 13.c

All the following would be considered current assets except A) marketable securities. B) cash. C) inventory. D) a warehouse.

C) interest rate risk. Explanation One disadvantage a bond mutual fund has is that there is no maturity date. At maturity, a bond returns the principal, regardless of what current market interest rates are. That is why as the bond approaches maturity, the interest rate risk declines (bonds with a shorter duration have less interest rate risk). The bond fund itself does not have that advantage because the fund does not mature. The risk of default is probably less with a bond fund because of the greater diversification than most individual portfolios can attain. The redemption at NAV feature of the mutual fund offers a liquidity that may not be available with some inactively traded individual bonds. Purchasing power risk would likely be similar in both cases. LO 8.i

Although investing in mutual funds has many advantages, there are some risks. One risk that is generally greater with a bond fund than a portfolio of individual bonds is A) liquidity risk. B) purchasing power risk. C) interest rate risk. D) default risk.

B) an accounting measure used to determine the contract owner's interest in the separate account. Explanation When money is deposited into the annuity, it is purchasing accumulation units. LO 9.b

An accumulation unit in a variable annuity contract is A) none of these. B) an accounting measure used to determine the contract owner's interest in the separate account. C) fixed in value until the holder retires. D) an accounting measure used to determine payments to the owner of the variable annuity.

D) II and III Explanation Joint account owners share ownership of the account and must be adults. A minor may not legally exercise control over an account and may not be an owner of record of an account. LO 1.b

An agent may open a joint account for which of the following? Lee and his 13-year-old son, Tom Mary and Kelley, two adult college roommates Jerry and Mark, friends and partners in business for more than 20 years Melinda and her minor nephew, John, for whom she is guardian A) I and III B) II and IV C) I and IV D) II and III

B) Purchase shares of an equity REIT Explanation Equity real estate investment trusts (REITs) are a way to have an ownership interest in real estate without having to manage properties or worry about collecting rents. REITs trade on exchanges and over the counter; therefore, they are liquid investments. Both of these characteristics meet the investor's objective and make equity REITs the most suitable recommendation of those offered here. A mortgage REIT does not own real estate, so it will not meet the investor's objective. LO 11.a

An experienced investor wants to allocate 10% of an existing portfolio to owning real estate but does not want to maintain properties, be a landlord, or wait if cash is needed. Which of the following choices would be suitable, given the investor's objectives? A) Purchase property and assign a management company B) Purchase shares of an equity REIT C) Purchase shares of a mortgage REIT D) Purchase shares of mortgage companies

A) changes in common stock prices tend to be more closely related to changes in the cost of living than changes in bond prices Explanation Because common stocks are not fixed-dollar investments, they have the opportunity to keep pace with inflation. LO 9.a

An important basic characteristic of common stocks that makes them a suitable type of investment for the separate account of variable annuities is A) changes in common stock prices tend to be more closely related to changes in the cost of living than changes in bond prices. B) the yield is always higher than bond yields. C) the safety of the principal invested. D) the yield is always higher than mortgage yields.

A) changes in common stock prices tend to be more closely related to changes in the cost of living than changes in bond prices. Explanation Because common stocks are not fixed-dollar investments, they have the opportunity to keep pace with inflation. LO 9.a

An important basic characteristic of common stocks that makes them a suitable type of investment for the separate account of variable annuities is A)changes in common stock prices tend to be more closely related to changes in the cost of living than changes in bond prices. B) the yield is always higher than bond yields. C) the safety of the principal invested. D) the yield is always higher than mortgage yields.

C) make delivery of the certificates accompanied by evidence of insurance, either on the face of the certificates or in a separate document. Explanation Although it is likely that the confirmation would include a statement that the bonds are insured, it is also necessary to provide the client with some proof of that insurance, either on the bond itself or, in the case of book entry delivery, as a separate document. LO 6.d

An insured municipal bond is purchased by your client in the secondary market. After the sale, Municipal Securities Rulemaking Board rules would require you to A) send a copy of the official statement. B) include a copy of the insurance policy with delivery of the certificates. C) make delivery of the certificates accompanied by evidence of insurance, either on the face of the certificates or in a separate document. D) indicate that the bonds are insured on the confirmation because this is the only requirement.

C) standby underwriting. Explanation In many cases, when a corporation is issuing new shares, existing shareholders receive preemptive or stock rights to buy these new shares to maintain their current proportionate ownership. In the event some of the rights are not used, the standby underwriter agrees to purchase those unsubscribed for shares. LO 3.f

An investment banker purchasing what is left unsold from a rights offering is engaging in A) all or none underwriting. B) firm commitment underwriting. C) standby underwriting. D) preemptive rights underwriting.

C) $320 Explanation The maximum gain on a credit spread is the net credit received (3.30 − 0.10 = 3.20 × 100 shares = $320). LO 10.e

An investor opens the following position: Write 1 CDE Oct 30 call at 3.30 Buy 1 CDE Oct 40 call at 0.10 The maximum gain is A) unlimited. B) $680. C) $320. D) $1,000.

D) the investor in the variable annuity contract reports no taxable consequences during the accumulation period. Explanation One of the features of annuities is the tax deferral of all earnings until the money is withdrawn. Mutual fund distributions are taxable when received. On the other hand, when the annuity accumulation is withdrawn, everything above the cost basis is taxed as ordinary income (10% penalty if younger than 59½)—there is never any capital gains treatment with annuities. Variable annuities invariably have higher expense ratios than mutual funds with similar portfolios. Surrender charges are found with annuities. Do not confuse those with the conditional deferred sales charge (CDSC) applied to certain mutual fund share classes. LO 9.d

An investor begins a periodic payment deferred variable annuity purchase program. One respect in which this differs from purchasing a mutual fund is that A) the variable annuity contract will generally have lower expenses than the mutual fund. B) there is a minimum guaranteed return with the variable annuity, while there are no guarantees with the mutual fund. C) the mutual fund will generally have a surrender charge for early withdrawal and variable annuities only charge for surrender when annuitizing. D) the investor in the variable annuity contract reports no taxable consequences during the accumulation period.

B) Maximum loss = $425; maximum profit = $2,575; breakeven point = $25.75 per shareExplanation The initial action is a $425 purchase. That debit is the most the investor can lose. To figure maximum profit, you must think! What kind of strategy is buying a put option—bearish! The investor anticipates a falling market. The maximum profit will be realized when the market price of the stock falls as low as it can, which is zero. If the market price does fall to zero, the investor can sell the put for its intrinsic value. In this case, the 30 put would be in the money by 30 points, or $3,000. Therefore, the maximum profit to the put buyer is the entire difference between the strike price and zero, offset (reduced) by the premium paid (strike price minus premium). Breakeven uses the put-down rule. You subtract the premium from the strike price and that is $30 minus $4.25, which = $25.75. LO 10.h

An investor goes long an XYZ May 30 put for 4¼ points. What is the investor's maximum gain, maximum loss, and breakeven point? A) Maximum loss = $425, maximum profit = $2,575; breakeven point is $34.25 per share. B) Maximum loss = $425; maximum profit = $2,575; breakeven point = $25.75 per share. C) Maximum loss is unlimited; maximum profit is $2,575; breakeven point = $25.75 per share. D) Maximum loss is $425, maximum profit is unlimited; breakeven point is $25.75 per share.

C) 30 puts Explanation Buying puts is the most effective strategy to hedge the risk of decline in a stock portfolio's market value. To determine the number of option contracts necessary to hedge, divide the portfolio value ($1.7 million) by the market value of the index (68,000). Multiply the result (25) by the beta of 1.20. The result is 30 contracts. LO 10.g

An investor has a diversified portfolio of common stock with a market value of $1.7 million and a beta of 1.20. If the OEX (S&P 100) is currently quoted at 680, to protect the portfolio against a decline in value, the investor's best strategy is to buy A) 25 calls. B) 25 puts. C) 30 puts. D) 30 calls.

B) II or III Explanation There are two investment strategies on the exam that can lead to a potentially unlimited loss. A short stock position gives an investor unlimited risk potential if the stock should rise because the investor must eventually buy back the stock at the higher price. Because stock can rise an unlimited amount, there is unlimited risk. The sale of a naked call requires that, if exercised, the writer must buy the stock in the market and deliver it at the strike price. LO 10.h

An investor holding which of the following positions is exposed to a potentially unlimited loss? Short 1 IBS Jul 50 put Short 100 shares of IBS stock Short 1 IBS Jul 50 uncovered call Short 1 IBS Jul 50 covered call A) I or II B) II or III C) II or IV D) I or III

C) the likelihood of recapture Explanation Recapture of deductions is a concern when accelerated, but not when straight-line depreciation is used. In any business, there is always concern about the quality of the management. By and large, DPPs are not liquid investments, so an investor needing a quick sale may have problems. The nature of DPPs tends to make them more sensitive to legislative risk than most other securities. LO 11.g

An investor in an equipment-leasing direct participation program (DPP) using straight-line depreciation would probably not be concerned about A) the quality of the management. B) legislative risk. C) the likelihood of recapture. D) liquidity risk.

C) $3,000 Explanation The maximum deduction of net capital losses against other income in any one year is $3,000; any remaining loss can be carried forward into the next year. LO 3.i

An investor in the 28% tax bracket has a $5,000 loss after netting all capital gains and losses realized. How much may the investor deduct from income that year? A) $5,000 B) $0 C) $3,000 D) $2,500

A) Oil and gas limited partnership interests Explanation The structure of a limited partnership does not allow for the payment of dividends. If there is income, it flows through to the investor, but it is not considered a dividend. Common stock can pay dividends and preferred stock is purchased for its dividend payout. UITs pay dividends in a manner similar to mutual funds. LO 11.f

An investor in which of the following products may not receive dividends? A) Oil and gas limited partnership interests B) Shares of common stock C) Units in a UIT D) Shares of preferred stock

B) Class C shares Explanation Class C shares are referred to as level load because the charges never fluctuate. Shares are purchased at net asset value (no front-end load), and there is a back-end load (CDSC), generally for 12 months. The 12b-1 fees on Class C shares are higher than on Class A shares and remain so until the position is liquidated. Class C shares are most suitable for investors who will not maintain the position for the long term. LO 8.d

An investor mentions the term level load fund shares to a registered representative. The investor is referring to A) shares of a closed-end fund. B) Class C shares. C) Class A shares. D) Class B shares.

B) receives $14,000 Explanation When an investor takes a long position in an option, it means that the investor has purchased the option. When a put is exercised, the holder must deliver the stock on settlement date. At that time, proceeds representing the strike price ($140) for 100 shares ($14,000) are received. LO 10.a

An investor opens a long position in one XYZ Nov 140 put @7. Disregarding any commissions, if the option is exercised, on settlement date the investor A) receives $700. B) receives $14,000. C) must pay $700. D) must pay $14,000.

C) Maximum gain is unlimited; maximum loss is $500; breakeven points are $35 and $45. Explanation The first step is to identify the position. This is a long straddle—a long put and a long call with identical terms. That means we are going to have two breakeven points. The maximum gain is unlimited because one of the positions is a long call. The maximum loss is the amount paid for the straddle (the two premiums totaling $500). Breakeven follows the call-up and put-down rules. Add the premium to the strike of the call ($40 + $5 = $45) and subtract the premium from the strike of the put ($40 ‒ $5 = $35). LO 10.h

An investor opens the following options position: Buy 1 FOZ Mar 40 call @3; buy 1 FOZ Mar 40 put @2. What is the investor's maximum gain, maximum loss, and breakeven point? A) Maximum gain is unlimited; maximum loss is $500; breakeven point is $40. B) Maximum gain is $500; maximum loss is unlimited; breakeven point is $40. C) Maximum gain is unlimited; maximum loss is $500; breakeven points are $35 and $45. D) Maximum gain is $3,500; maximum loss is $500; breakeven points are $35 and $45.

D) Maximum gain = $375; maximum loss = $125; breakeven point = $48.75. Explanation The first step is to identify the position. This is a credit call spread. It is a credit spread because the option sold brought in a higher premium than the one purchased. The credit of $375 is the most the investor can make. This is a bear call spread. We know that because the investor purchased the option with the higher strike price and sold the one with the lower strike price. The goal is for the stock's price to decline to the point where neither option is expired. That way, the investor keeps the net credit. If the market does not cooperate and the stock rises above $50 per share, the 45 call will be exercised and the investor will be required to sell the stock for $4,500. However, the stock needed for delivery can be obtained by exercising the long position for $5,000. This results in a loss of $500. When the $375 credit received is taken into consideration, the loss of $500 is reduced by the $375 premium, resulting in a maximum loss of $125. The quick way to do this is to subtract the net premium (the $375 credit) from the difference in strike prices (5 points) and the result is the same $125 loss. Breakeven follows the call-up rule; add the net credit to the lower strike price and that is $3.75 + $45 = $48.75. LO 10.h

An investor opens the following options position: Long 1 KAP Jul 50 call @ 4½ and short 1 KAP Jul 45 call @8¼. What is the investor's maximum gain, maximum loss, and breakeven point? A) Maximum gain = $125; maximum loss = $375; breakeven point = $48.75. B) Maximum gain = $125; maximum loss = $375; breakeven point = $46.25. C) Maximum gain = $375; maximum loss = $125; breakeven point = $46.25. D) Maximum gain = $375; maximum loss = $125; breakeven point = $48.75.

D) Maximum gain is $225; maximum loss is $275; breakeven point is $47.25. Explanation The first step is to identify the position. This is a debit put spread. It is a debit spread because the option purchased cost more than the one sold. The debit of $275 is the most the investor can lose. This is a bear put spread. We know that because the investor purchased the option with the higher strike price and sold the one with the lower strike price. The goal is for the stock's price to decline to the point where both options are exercised. For example, if the market price of RAV should fall below 45, the owner of the 45 put will exercise, causing the seller to purchase the stock for $4,500. The seller can then exercise the long 50 put and deliver the stock purchased at 45 for 50. That is a profit of $500 less the cost of the options (the debit of $275). The breakeven point follows the put-down rule. Subtract the net premium (the $2.75 debit) from the higher strike price resulting in a breakeven point at $47.25. LO 10.h

An investor opens the following options position: Long 1 RAV Mar 50 put @5¾; short 1 RAV Mar 45 put @3. What is the investor's maximum gain, maximum loss, and breakeven point? A) Maximum gain is $225; maximum loss is $275; breakeven point is $47.75. B) Maximum gain is $275; maximum loss is $225; breakeven point is $47.25. C) Maximum gain is $275; maximum loss is $225; breakeven point is $47.75. D) Maximum gain is $225; maximum loss is $275; breakeven point is $47.25.

A) Maximum gain is $525; maximum loss is unlimited; breakeven points are $24.75 and $40.25. Explanation The first step is to identify the position. This is a short combination; a short put and a short call with different terms. That means we are going to have two breakeven points. The maximum loss is unlimited because one of the positions is an uncovered call. The maximum profit is the premiums (credit) received of $525. Breakeven points follow the call-up and put-down rule. That is, add the premiums of $5¼ to the strike price of the call ($35 + $5.25= $40.25) and subtract the premiums of $5¼ from the strike price of the put ($30 ‒ $5.25 = $24.75). LO 10.h

An investor opens the following options position: Write 1 RAN Dec 35 call @4; short 1 RAN Dec 30 put @1¼. What is the investor's maximum gain, maximum loss, and breakeven point? A) Maximum gain is $525; maximum loss is unlimited; breakeven points are $24.75 and $40.25. B) Maximum gain is $525; maximum loss is $2,975; breakeven points are $24.75 and $40.25. C) Maximum gain is $525; maximum loss is unlimited; breakeven point is $32.50. D) Maximum gain is unlimited; maximum loss is $525; breakeven points are $24.75 and $40.25.

A) $440 Explanation The maximum loss on a debit spread is the net debit. LO 10.e

An investor opens the following position: Buy 1 COD Jan 40 put at 6.50 Write 1 COD Jan 30 put at 2.10 His maximum loss is A) $440. B) $2,600. C) $560. D) $2,100.

A) $2,300 short-term capital gain Explanation The purchase of the put on stock that had a holding period of less than long term (March to January is only 10 months) erases the holding period. The holding period begins again once the put expires. In this case, we have a holding period from expiration in April until June 2. That is clearly a short-term holding period. As far as the math, the cost is $62 per share ($60 cost of the stock plus $2 for the put). The sale price was $85, the gain is $23 per share, $2,300. Alternatively, the sale price of $85 created a $2,500 gain (85-60) and the put was a $200 loss. That is a net of + $2,300. Either way is fine. LO 10.i

An investor purchased 100 shares of ABC common stock at $60 per share on March 2, 2019. With the stock selling at $80 per share on January 2, 2020, the investor purchased an ABC Apr 75 put for a premium of 2. On June 2, 2020, the investor sold the stock for $85 per share. As a result, the tax consequences are A) $2,300 short-term capital gain. B) $2,500 short-term capital gain. C) $2,500 long-term capital gain. D) $2,300 long-term capital gain.

A) $500 long-term capital gain Explanation Let's take the positions one at a time. The stock is purchased for $83 and sold for $90. That is a $700 long-term gain (Oct 2020 to Nov 2021). The option is sold for $300 and closed (bought back) at the $5 intrinsic value (the difference between the 85 strike and the 90 market) for a short-term capital loss of $200. The net of the two (+700 and -200) is a $500 long-term capital gain. If you thought that writing the call when the stock's holding period was still short-term had an impact, you were thinking of the effect of buying a put on stock held short-term. Buying or selling a call does not affect the holding period of the underlying stock. LO 10.i

An investor purchased 100 shares of Paradigm Publishing Corporation (PPC) on October 17, 2020. The price was $83 per share. On April 11, 2021, the investor wrote one PPC Nov 85 call for 3. At expiration date, the PPC stock is selling for $90 per share, and the investor liquidates the stock at the market price and closes the option at its intrinsic value. The net tax consequences are A) $500 long-term capital gain. B) $500 short-term capital gain. C) $200 long-term capital gain. D) $900 long-term capital gain.

A) $47 per share. Explanation This is a wash sale situation. Selling a stock at a loss and repurchasing it within 30 days "washes" out the loss for current tax purposes. The loss, in this case $3 per share, is added to the cost of the repurchased stock. Thus, $44 plus $3 equals a new cost basis of $47 per share. LO 13.h

An investor purchased 100 shares of a stock three years ago at $38 per share. Disappointed with the stock's performance, the investor sells it for $35 per share. Two weeks later, after the company announced higher-than-expected earnings, the investor purchased 100 shares at $44 per share. When this investor decides to sell the newly purchased shares, the cost basis will be A) $47 per share. B) $38 per share. C) $41 per share. D) $44 per share.

A) a capital gain Explanation Because the investor sold the bond at a price that will yield less than the yield when she purchased the bond, the bond must have been sold for more than the investor paid for it. Therefore, the investor profited by that difference. Remember, the higher the price, the lower the yield. LO 6.f

An investor purchased a municipal bond at par to yield 5.5% to maturity. If, two years later, she sold the bond at a price equivalent to a 5% yield to maturity, the investor incurred A) a capital gain. B) a capital loss. C) taxable interest income. D) no taxable result at this time.

B) $157 per share Explanation This investor is looking for the price to go up. The purchase price was $153 and the cost of the "insurance" (the put option) was 4. That means that the investor will not start making money until the stock rises above the cost of the stock plus the cost of the put ($153 plus $4 = $157). In a question like this, the current market price of the stock and the exercise price of the option are irrelevant. Breakeven on long stock and long put is the cost of the stock plus the cost of the put. As is always the case when computing breakeven, the number of shares and number of option contracts is meaningless−breakeven is the same price for one or one thousand. LO 10.h

An investor purchases 1,000 shares of PLEX common stock at a price of $153 per share. Shortly afterward, with PLEX selling for 149 per share, a purchase of 10 PLEX 150 puts at 4 takes place. What is the investor's breakeven point? A) $149 per share B) $157 per share C) $113 per share D) $153 per share

D) I and III Explanation The wash sale rule is violated when an investor sells a security at a loss and purchases the same or a substantially identical security within 30 days of the sale date. The IRS considers a call option substantially identical to the underlying stock because it represents the right to buy the shares. LO 13.h

An investor purchases 100 shares of XYZ common stock for $70 and sells it one year later for $50. Which of the following activities would violate the wash sale rule? Purchasing an XYZ call option 20 days after the sale Purchasing an XYZ put option 20 days after the sale Purchasing 100 shares of XYZ common stock 20 days after the sale Selling short 100 shares of XYZ common stock 20 days after the sale A) II and IV B) I and II C) III and IV D) I and III

D) On the settlement date, the investor will receive cash in the amount of the intrinsic value. Explanation Yield-based options settle in cash. The settlement amount is the intrinsic value. When an investor holding a put option exercises, the debt security is, in essence, being sold to (put to) the writer of the option. Instead of physical delivery of the bonds, settlement is made in cash. It is quite possible the investor will simply sell the option for its intrinsic value. That isn't the correct answer to this question because you are told the investor exercises the put. LO 10.g

An investor purchases a U.S. Treasury bond put option. The option is in the money on the last trading day and the investor exercises the put. Which of the following statements is correct? A) On the settlement date, the investor will deliver the bonds and receive cash in the amount of the intrinsic value. B) On the settlement date, the investor will pay cash in the amount of the intrinsic value. C) The investor will sell the option for its intrinsic value. D) On the settlement date, the investor will receive cash in the amount of the intrinsic value.

C) a calendar spread. Explanation A calendar (or time or horizontal) spread is one where the options are of the same type (long and short puts or long and short calls) on the same security, but with different expiration dates. We know this is not a debit spread because the option October expiration has more time value than the July expiration. Therefore, the option sold (Oct) will have a higher premium than the one bought resulting in a credit. Although a combination involves different expiration months, it is either two longs or two shorts, not a long and a short. LO 10.e

An investor purchases one ABC Jul 50 put and sells one ABC Oct 50 put. This strategy is A) a debit spread. B) a combination. C) a calendar spread. D) a diagonal spread.

B) AA-rated mortgage bonds.Explanation The conservative level of risk eliminates the income bonds and the junk bonds. Income bonds pay interest only if the issuer has the funds to do so. Junk bonds are named such because of their high risk. Even though the convertible debentures have a higher rating than the mortgage bonds, the difference is relatively insignificant at that level and either would be suitable for the conservative investor. However, because of the convertible feature, it is always true on the exam that the income return is lower than non-convertible issues. Therefore, the most suitable for this investor would be the mortgage bonds.

An investor seeking income combined with a conservative level of risk would purchase A) AAA-rated convertible debentures. B) AA-rated mortgage bonds. C) unrated income bonds. D) junk bonds.

A) Zero-tranche collateralized mortgage obligation (CMO) with an estimated five years of life Explanation A zero-tranche CMO is subject to interest rate risk as well as extension risk when interest rates rise, and therefore, it would not be suitable for a customer that needs her investment back at a specific point in the future. By contrast, a four-year zero coupon bond will mature within the anticipated time frame for needing the funds and would be the most suitable choice of the answers given. LO 12.d

An investor wants to invest $20,000 but anticipates needing those funds in five years for a business investment. Currently, with inflation rising, the government is expected to take action to push interest rates up to reduce the money supply. Given these conditions, which of the following securities would be the least suitable for this investor who needs a specific amount of money in five years? A) Zero-tranche collateralized mortgage obligation (CMO) with an estimated five years of life B) Corporate bonds maturing in five years C) Zero-coupon bond maturing in four years D) U.S. Treasury bonds maturing in six years

C) Income bond, convertible bond, nonconvertible bond Explanation The income (or adjustment) bond is the least suitable because it is issued by companies coming out of bankruptcy with interest payable only if the money is available. Therefore, it is not suitable given the objective. A convertible bond has a lower coupon than a nonconvertible bond because of the convertibility feature. Therefore, if seeking to maximize income, the corporate bond would be the most suitable of the three choices (from least to most: income bond, convertible bond, and nonconvertible bond). LO 5.a

An investor wants to maximize income using debt securities. Which of the following lists rank securities from the least suitable to the most suitable recommendation if income is the investment objective? A) Treasury bills, convertible bond, income bond B) Nonconvertible bond, convertible bond, income bond C) Income bond, convertible bond, nonconvertible bond D) Convertible bond, income bond, nonconvertible bond

C) $450 loss Explanation The investor exercised the right to buy the stock for 75 and can sell the stock in the market for 77, for a gain of 2. The investor paid a premium of 6.50 minus the gain of 2, which gives the investor a loss of 4.50 (4.50 × 100 = $450). LO 10.h

An investor with no other positions buys 1 DWQ May 75 call at 6.50. If the investor exercises the call when the stock is trading at 77 and immediately sells the stock in the market, what is the investor's profit or loss? A)$350 profit B)$350 loss C) $450 loss D) $450 profit

C) regulatory risk Explanation Regulatory risk is generally not associated with investing in CMOs. All of the other risks are associated with CMOs. Extension risk is the uncertainty that the mortgages will be paid off later than expected. This typically happens when interest rates rise. After all, who is going to refinance a mortgage at a higher rate? Prepayment risk is just the opposite; the mortgages might be paid off more quickly and the income stream will cease. This typically happens when interest rates decline, but they are also factor in people moving and selling their homes. CMOs are subject to interest rate risk just like other debt securities. LO 12.d

An investor would assume all of the following risks when investing in a collateralized mortgage obligation (CMO) except A) prepayment risk. B) extension risk. C) regulatory risk. D) interest rate risk.

D) because he is bullish. Explanation A put seller (writer) will benefit as long as the market price of the underlying security does not drop substantially. His position is bullish. LO 10.d

An investor would sell a put A) because he is bearish. B) as a substitute for a short sale. C) as an inflation hedge. D) because he is bullish.

C) a $185 profit Explanation The opening sale of the IBS put was made for 16.60, and the closing purchase was made for the intrinsic value of 14.75. The put's intrinsic value is determined by how far the stock's market price is below the strike price. (In this case, 280 minus 265.25.) 16.60 − 14.75 = 1.85 × 100 shares = $185.00. The investor profits because the sale's proceeds exceed the purchase price. LO 10.h

An investor writes 1 IBS 280 put for 16.60. The position is closed, and the put is bought for its intrinsic value when IBS is trading at 265.25. The investor realizes A) a $145 profit. B) a $185 loss. C) a $185 profit. D) a $235 loss.

C) long straddles Explanation The contracts will not be exercised if options expire at the money. Therefore, writers will show a profit, but buyers will not. LO 10.f

At expiration, if the market price of the underlying common stock and the strike price are the same, all of the following customer positions will show a profit except A) short straddles. B) short calls. C) long straddles. D) short puts.

C) $0.01. Explanation Automatic exercise will occur for equity options at expiration that are in the money by at least 0.01, unless specific instructions are given by the customer not to do so. LO 10.j

Automatic exercise will occur for equity options at expiration that are in the money by at least A) one-eighth of a point. B) one-quarter of a point. C) $0.01. D) $0.05.

A) sold Explanation Depletion allowances are allowed to compensate for a mineral resource, which is considered accomplished when it is sold. LO 11.f

Depletion allowances in oil and gas programs are based on the amount of oil A) sold. B) in reserve. C) lost to shrinkage. D) extracted.

D) when the price for each purchase is the same. Explanation There are two requirements for a dollar cost averaging program to work. The first is that the same amount must be invested at each specified interval. The second is that the price per transaction does not remain the same. If that is the case, then the average cost per share and average price paid per transaction are the same. The price needs to move for DCA to show a benefit. LO 8.e

Dollar cost averaging (DCA) will always result in a lower cost per share than the price paid per share except A) when the price for each purchase is increasing. B) when the price for each purchase is decreasing. C) when the price for each purchase is fluctuating.. D) when the price for each purchase is the same.

C) Planned amortization class CMOs (PACs) Explanation PACs are designed to reduce both extension and prepayment risk by transferring those risks to a companion tranche. That tranche is sometimes called a support tranche and it absorbs those risks. TACs also have companion tranches, but they only absorb the prepayment risk. LO 12.d

Extension risk and prepayment risk are among the most prominent risks facing investors in collateralized mortgage obligations (CMOs). That risk would likely be the least in which of the following? A) Target amortization class CMOs (TACs) B) Companion or support tranches C) Planned amortization class CMOs (PACs) D) Plain vanilla CMOs

A) Specialized Explanation A specialized or sector fund invests 25% or more of its assets in a particular region or industry. LO 8.g

If an investor wants to invest in the electronics industry but does not want to limit his investments to one or two companies, which type of fund would be most suitable? A) Specialized B) Hedge C) Money market D) Bond

D) Designating to FINRA an anti-money laundering compliance officer Explanation FINRA's Rule 3130 on anti-money laundering requires members to designate to FINRA an individual or individuals responsible for implementing and monitoring the day-to-day operations and internal controls of the program. The AML program is neither filed with nor approved by the SEC or FINRA. However, when an examiner from either of these bodies pays a surprise visit to the firm, there had better be a well-detailed program available to show. There is no specific schedule for updating. Updating and training of personnel is an ongoing project. LO 15.f

FINRA requires a member firm to develop, implement, and monitor anti-money laundering programs designed to achieve compliance the Bank Secrecy Act and related regulations. Which of the following is required of a broker-dealer's anti-money laundering program? A) Filing of the firm's AML program with FINRA B) Updating the program every 36 months C) Approval of the firm's AML program by the SEC D) Designating to FINRA an anti-money laundering compliance officer

D) I and IV Explanation 12b-1 fees may be used only to cover promotional and other distribution expenses for funds that are distributors of their own shares; fee amounts must be disclosed in the prospectus. The fund may not use the term no load in any communications with the public if the 12b-1 fee and other service fees exceed 0.25% of average net assets. LO 8.d

For a mutual fund that collects a 12b-1 fee, which of the following statements are true? The fund may use the money to pay for mailing sales literature. Advertising materials must always state that the fund is no load. The fund may use the money to pay for commissions on portfolio transactions. The fund's prospectus must disclose the fee. A) II and III B) III and IV C) I and II D) I and IV

A) equal to the option premium Explanation An option premium consists of two factors. First is the intrinsic value (the amount the option is in the money). The balance of the premium represents the time value. An out-of-the-money option, by definition, has no intrinsic value. Therefore, the entire premium represents time value. The time value of an option will never be less than zero; it can only be zero or positive. LO 10.c

For an out-of-the-money equity option expiring in seven months, the time value of the option will most likely be A) equal to the option premium. B) greater than the option premium. C) less than zero. D) less than the option premium.

C) A DPP Explanation When it comes to liquidity, at least for the exam, DPPs rank near the bottom of the list. Mutual funds and UITs are redeemable by the issuer, and closing a position in an option contract settles the next day. LO 11.g

From time to time, an investor's situation arises where they may need to liquidate a portion of the portfolio. It could be a medical need, an emergency repair, or a joyous event such as a wedding. Getting the necessary cash would be most difficult from which of the following holdings? A) A mutual fund B) A listed option C) A DPP D) A unit investment trust

D) one contract for 105 shares with an effective exercise price of $28.57. Explanation When a company pays a stock dividend or effects a fractional stock split, the underlying option is adjusted by increasing the number of shares the contract covers (5% × 100 shares = 5 additional shares). The number of contracts owned remains the same, and the effective exercise price is adjusted so that the position value before and after the adjustment remains the same ($3,000 / 105 = $28.57). LO 10.j

If a 5% stock dividend is declared, the owner of 1 XYZ Jul 30 call owns A) one contract for 100 shares with an exercise price of 30. B) two contracts for 100 shares with an exercise price of 30. C) two contracts for 105 shares with an exercise price of 30. D) one contract for 105 shares with an effective exercise price of $28.57.

A) buy Japanese yen calls Explanation The Japanese exporter will be paid in U.S. dollars upon delivery of the equipment. He would be adversely affected if the dollar dropped in value in relation to the yen. To protect his position, he should buy calls on his own currency—the yen. Then if the yen appreciates, his loss on the dollar is offset by his gain on the calls. Exporters buy puts on foreign currency to hedge, but there are no options on the U.S. dollar. The next best strategy is to buy calls on the home currency. LO 10.g

If a Japanese exporter wants to hedge a recent sale of stereo equipment to a U.S. buyer, and the exporter will be paid in U.S. dollars upon delivery of the goods, the best hedge would be to A) buy Japanese yen calls. B) buy Japanese yen puts. C) sell Japanese yen puts. D) sell Japanese yen calls.

D) II and III Explanation Buying puts and writing calls are bearish strategies. LO 10.d

If a customer believes the price of ABC is going to fall, which of the following option strategies would be appropriate? Buy calls on ABC. Write calls on ABC. Buy puts on ABC. Write puts on ABC. A) I and IV B) I and II C) II and IV D) II and III

C) unlimited Explanation Maximum gain in a long straddle is unlimited if the market moves up. If the market moves to zero, the gain is $4,400 (50 − 6 = 44). LO 10.f

If a customer buys 1 ABC Jan 50 call at 2 and 1 ABC Jan 50 put at 4 when ABC is at 49, the maximum potential gain is A) $200. B) $400. C) unlimited. D) $600.

C) $800. Explanation This is a credit spread. (More premium was received than was paid.) The maximum gain to a seller is the premium received (net credit of 7). In a spread, the maximum gain plus the maximum loss equals the difference in strike prices (75 − 60 = 15). Therefore, 15 minus the maximum gain of 7 equals the maximum loss of 8 multiplied by $100, or $800. LO 10.e

If a customer buys 1 ABC Jan 60 put at 6 and writes 1 ABC Jan 75 put at 13, the maximum loss is A) $1,500. B) $700. C) $800. D) $900.

D) $5,300 Explanation The cost basis of the 100 shares is the total amount the investor spent to acquire them. She paid $300 to purchase the call option. When she exercised the call, she purchased 100 shares of FLB at $50 per share for $5,000, so the cost basis is $5,300. LO 10.i

If a customer buys 1 FLB Oct 50 call at 3 and she exercises the option to buy 100 shares when the market is at 60, what is the cost basis of the 100 shares? A) $5,000 B) $6,300 C) $6,000 D) $5,300

B) the cost of stock purchased less premium Explanation When the investor owns stock and sells a call, the call is covered. Breakeven is computed by subtracting the premium from the stock's purchase price. LO 10.d

If a customer buys 100 shares of stock and writes one out-of-the-money call against her long position, the breakeven point is A) the strike price less premium. B) the cost of stock purchased less premium. C) the strike price plus premium. D) the cost of stock purchased plus premium.

A) the lower of either the yield to call or yield to maturity Explanation MSRB Rule G-15 states that the confirmation must indicate the lowest possible yield. LO 15.a

If a customer buys callable municipal bonds, Municipal Securities Rulemaking Board (MSRB) rules state that the confirmation sent to the customer must disclose A) the lower of either the yield to call or yield to maturity. B) the higher of either the yield to call or yield to maturity. C) the nominal yield only. D) the yield that would result if the bonds were called midway between the date they become eligible to be called and their maturity date.

D) the next time the fund computes it. Explanation Orders to redeem shares are executed at the next computed price. This is an example of the forward pricing rule. LO 8.c

If a customer submits a redemption order to her broker-dealer after the close of the New York Stock Exchange, she will receive a price based on the net asset value computed A) within the next two business days. B) the same day, regardless of when the order is received. C) the previous business day. D) the next time the fund computes it.

A) a cash account with full power of attorney Explanation For anyone other than the account owner, entering trades and withdrawing assets requires a full power of attorney. A limited power of attorney enables a nonaccount owner to enter trades but not to withdraw assets. LO 1.d

If a customer wishes to open a cash account in her name only and allow her spouse to make purchases and receive checks in his name only, she must instruct her broker-dealer to open A) a cash account with full power of attorney. B) a margin account. C) a cash account. D) a cash account with limited power of attorney.

D) He pays no estate tax. Explanation Establishing a JTWROS account allows for the transfer of assets to the survivor upon death. The surviving spouse is not taxed on assets transferred in this manner because under current tax law, there is an unlimited marital deduction. LO 13.h

If a married couple establishes a joint tenants with right of survivorship (JTWROS) account with a balance of $1 million and the wife dies, what is the husband's estate tax liability? A) He pays federal taxes only on $500,000. B) He pays federal and state taxes on the entire balance. C) He pays federal and state taxes on $500,000. D) He pays no estate tax.

B) a common stock fund with a low portfolio turnover. Explanation Of the choices given, common stock is the only vehicle capable of providing long-term growth. Preferred stock will provide dividends, but it will not provide much growth, as it trades like a bond in line with interest rate changes. Of the two common stock funds, the one with the lower portfolio turnover will have lower annual expenses. LO 8.g

If a married couple with a long-term growth objective is considering a mutual fund, and they are concerned about the fund's annual expenses, they should select A) a preferred stock fund. B) a common stock fund with a low portfolio turnover. C) a long-term corporate bond fund. D) a common stock fund with a high portfolio turnover.

B)Comparing an equity growth fund to a money market fund, with the intention of convincing an investor to purchase the growth fund Explanation A characteristic of money market funds is that they deliberately avoid growth. Thus, for the growth investor, comparing a money market fund to a growth fund is unfair. LO 8.g

If a registered representative is comparing two mutual funds for a customer, which of the following comparisons would not be permissible? A) Comparing two equity funds with slightly different investment objectives, even if the differences and their consequences are carefully explained B) Comparing an equity growth fund to a money market fund, with the intention of convincing an investor to purchase the growth fund C) Comparing diversified growth funds from two different fund families D) Comparing a long-term bond fund to a shorter-term bond fund to demonstrate the trade-offs that exist between risk and return.

A) $14 Explanation The purchaser of a bond pays the seller the interest that has accrued since the last interest payment date. A purchaser in August will pay the interest that has accrued since May 1. Then, on November 1, the investor will receive the entire six months of interest. We are told that the investor paid $16 in accrued interest. That is income to the seller. Then, when the November payment of $30 (6% coupon is $30 semiannually) is made, the investor must report the amount over the accrued interest paid out as income. In our question, that is $30 minus $16 = $14. LO 6.e

If an M&N 1 corporate bond issued at par with a 6% coupon is later purchased in August for 97 plus accrued interest of $16, how much taxable interest must the investor report for the year? A) $14 B) $30 C) $60 D) $16

A) additional issues will have junior liens Explanation These additional issues are also known as junior lien bonds. Under a closed-end indenture, additional bonds issued against the same stream of revenues have a junior (subordinate) claim to those already outstanding unless the funds are required to complete construction of the facility. LO 6.b

If an indenture has a closed-end provision, this means A) additional issues will have junior liens. B) a sinking or surplus fund must be established. C) additional issues have no lien on the revenue stream. D) the bonds must be called before maturity.

B) credited the in-the-money amount. Explanation When index options are exercised, settlement is in cash rather than stock. The option writer delivers cash to the option buyer equal to the amount that the option is in the money. LO 10.g

If an index option is exercised, the holder's account will be A) debited the in-the-money amount. B) credited the in-the-money amount. C) credited the out-of-the money amount. D) debited the out-of-the money amount.

B) a unit investment trust Explanation A unit investment trust issues shares that represent units of a particular portfolio; management has no authority—or only limited authority—to change the portfolio. The portfolio is fixed, not traded. LO 8.a

If an investment company invests in a fixed portfolio of municipal or corporate bonds, it is classified as A) a growth fund. B) a unit investment trust. C) a closed-end company. D) a utilities fund.

D) Premium of 5 Explanation The OCC sets standard exercise prices and expiration dates for all listed options, but the options premiums that buyers pay are determined by the market. LO 10.b

If an investor buys 1 DWQ Apr 70 call at 5, giving him the right to buy 100 shares of DWQ at $70 per share, which aspect of the transaction is not set or standardized by the Options Clearing Corporation (OCC)? A) Contract size of 100 shares B) Expiration date in April C) Exercise price of 70 D) Premium of 5

A) capital gains rates on capital gains distributions and ordinary income rates on dividends. Explanation Capital gains distributions are taxed as capital gains, with their holding status depending on how long the fund has held the securities, not how long the investor has held the mutual fund shares. Dividend distributions are taxed as ordinary income. LO 8.f

If an investor has received dividends and capital gains distributions on mutual fund shares she has held for four months, she will pay A) capital gains rates on capital gains distributions and ordinary income rates on dividends. B) long-term or short-term capital gains rates, depending on the length of time the customer has held the fund shares. C) ordinary income tax rates on the capital gains and dividends. D) no tax until she liquidates the shares.

B) three points in the money Explanation The put is in the money when the underlying instrument's market price is below the put's strike price. LO 10.g

If the Swiss franc closes at 56, 1 SF 59 put is A) without intrinsic value. B) three points in the money. C) three points out of the money. D) at the money.

C) 6.25% Explanation To calculate the percentage yield of the underlying Treasury security, divide the strike price by 10 (62.50 / 10 = 6.25%). LO 10.g

If the strike price of a yield-based option is 62.50, this represents a yield of A) 0.00625%. B) 0.625%. C) 6.25%. D) 0.0625%.

A) It is halted. Explanation Options trading is always halted when the trading of the underlying security is halted. Options rely primarily on the underlying market value for premium determination. LO 10.j

If trading is halted in a listed stock, what happens to the trading in the stock's listed options? A) It is halted. B) It increases. C) It continues to trade. D) It is restricted to closing transactions.

D) I and III Explanation Combinations and straddles have two breakeven points. To calculate these breakeven points, add the combined premiums to the call strike price and subtract the combined premiums from the put strike price. LO 10.f

If your client writes a combination of a DWQ 45 call and a DWQ 50 put, and the premiums total $650, he breaks even when the price of the underlying stock is $43.50. $50.50. $51.50. $56.50. A) II and IV B) III and IV C) I and IV D) I and III

B) Accreted during the life of the bond and not taxed Explanation Under IRS rules, an owner of an original issue municipal discount bond must adjust the bond's cost basis by accreting the discount over the life of the bond. The accretion is not taxed. LO 6.f

If your customer bought an original issue discount bond from the Mount Vernon Port Authority, how is the discount on this bond taxed? A) Amortized during the life of the bond and not taxed B) Accreted during the life of the bond and not taxed C) As ordinary income D) As capital gains

D) An insured short-term municipal bond fund Explanation High-yield income funds usually invest in low-rated bonds, and income bonds do not pay interest unless the board of directors declares a payment. An insured short-term municipal bond mutual fund is relatively safe, very liquid, and provides income free from federal tax. LO 8.g

If your great-grandmother is interested in safety, liquidity, and tax-free income, which of the following would you recommend? A) Income bonds B) An exploratory oil and gas pool C) A high-yield income fund D) An insured short-term municipal bond fund

B) It is treated as a capital gain or loss Explanation All listed options trades with resulting gains or losses are treated as capital gains or losses. LO 10.i

In February, a customer sells 1 GHI Oct 60 put for 3 and buys 1 GHI Oct 70 put for 11. If the customer closes the Oct 70 put before expiration, which of the following statements regarding the resulting profit or loss is true? A) It is treated as ordinary income or loss. B) It is treated as a capital gain or loss. C) It depends on the disposition of the short Oct 60 put. D) It cannot be determined from the information given.

A) II and III Explanation This debit spread becomes profitable if the spread widens between the premiums. Credit spreads are profitable if the spread narrows between the premiums. If both puts are exercised, the spread is profitable. If the short 50 put is exercised, the customer buys the stock and sells it for 60 by exercising the long 60 put ($1,000 profit − $800 premiums = net $200 profit). LO 10.e

In March, a customer sells 1 ABC Oct 50 put for 3 and buys 1 ABC Oct 60 put for 11. The customer will experience a pretax profit from these positions if the difference between the premiums narrows to less than $8. the difference between the premiums widens to more than $8. both puts are exercised at the same time. both puts expire unexercised. A) II and III B) I and IV C) I and III D) II and IV

D) A stock with a beta of 0.5 Explanation Beta is a measure of a stock's volatility relative to the overall market, as measured by the S&P 500. A stock with a beta of 2.0 will move twice as fast as the overall market, while a stock with a beta of 0.5 will move half as fast as the overall market. LO 13.b

In a rising market, which of the following is least volatile? A) A stock with an alpha of +2.0 B) A stock with an alpha of +0.5 C) A stock with a beta of 2.0 D) A stock with a beta of 0.5

A) A closed-end investment company Explanation The capital structure of closed-end investment companies differs from other investment companies. Closed-end investment companies may issue debt securities, as well as preferred stock. Open-end companies and UITs can purchase debt securities for their portfolios but can only issue one class of equity. LO 8.b

In an effort to raise additional capital, which type of registered investment company may issue debt securities? A) A closed-end investment company B) An open-end investment company C) A face amount certificate company D) A unit investment trust

D) I and II Explanation An investor wants a debit spread to widen (I and II). As the difference between premiums increases, so does potential profit because the investor may sell the option with the higher premium and buy back the option with the lower premium. With credit spreads, investors profit if the spread between the premiums narrows. LO 10.e

In which of the following strategies would the investor want the spread to widen? Buy 1 RST May 30 put, write 1 RST May 25 put Write 1 RST Apr 45 put, buy 1 RST Apr 55 put Buy 1 RST Nov 65 put, write 1 RST Nov 75 put Buy 1 RST Jan 40 call, write 1 RST Jan 30 call A) I and IV B) III and IV C) II and III D) I and II

D)Treasury bonds Explanation Interest on Treasury bonds is not taxed at the state or local level. LO 7.e

Income from all of the following securities is fully taxable at the federal, state, and local levels except A) corporate bonds. B) Ginnie Maes. C) reinvested mutual fund dividends. D) Treasury bonds.

A) Convertible bonds Explanation Fifty percent of dividend income received from investments in common stock and preferred stock is excluded from taxation for a corporate investor. This exclusion applies to dividends from mutual funds where all of the portfolio securities are preferred or common stock. LO 13.g

Income from which of the following is not partially exempt to a corporate investor? A) Convertible bonds B) Preferred stock mutual funds C) Preferred stock D) Common stock

D) Common stock shares of public utility companies Explanation Interest rate risk affects the shares of public utility common stock in two ways. First, for most investors, public utility stocks are attractive because of their dividend yield. Therefore, if market interest rates rise, unless the utility can increases the dividend, the price of the stock will decline. That is where the second part comes into play. Public utilities are known for their highly leveraged capital structures. Put simply, they borrow a lot of money. An increase to market interest rates will likely cause their borrowing expenses to rise. With increased expenses, earnings fall and that can lead to a reduction to the dividend payout. The primary factor affecting the market price of shares of growth companies is the future expectations of earnings growth for these companies. Therefore, their market prices are not correlated with current interest rate changes. In addition, these companies rarely pay much, if any, dividend. As stated in the question, interest rate risk applies to companies paying income. Bankrupt companies do not pay dividends. High tech companies typically have very little debt in their capital structure and rarely pay dividends, so their common stock prices are not interest rate sensitive. LO 14.a

Interest rate risk is intrinsic to all types of fixed-income investments, including debt securities and preferred stock. When interest rates go up, market prices decline. Although not commonly associated with common stock, some common stock investments are subject to interest rate risk. The common stock of which of the following companies would be most affected by interest rate risk? A) Common stock shares of ABC High Tech Company B) Common stock shares of a company that has recently filed for bankruptcy C) Common stock shares of investment company growth funds D) Common stock shares of public utility companies

C) tax preference items Explanation Certain items receive favorable tax treatment from the IRS. One example is tax-exempt interest on private-purpose municipal revenue bonds. These types of items are known as tax preference items. For investors who are subject to the AMT, the benefits normally associated with tax preference items are lost because these items must be added back into the investor's taxable income. LO 6.f

Investors who are subject to the alternative minimum tax (AMT) will lose the tax benefits normally associated with A) gains associated with variable annuity portfolios. B) capital losses. C) tax preference items. D) losses on options positions.

A) an extraordinary call Explanation An extraordinary call can occur at an unknown point in time prior to maturity. The issuer has the right to call the bonds, generally at par, when a certain event occurs, typically a catastrophe. Because catastrophe calls are unplanned or unexpected, the term that will often be used is extraordinary. LO 6.b

It is not uncommon for municipal revenue bonds to have a catastrophe call provision. Another term that might be used to refer to this provision is A) an extraordinary call. B) a premium call. C) an unplanned call. D) an unexpected call.

C) a tender offer Explanation The SEC defines tender offer as "an active and widespread solicitation by a company or third party (often called the bidder or offeror) to purchase a substantial percentage of the company's securities. Bidders may conduct tender offers to acquire equity (common stock) in a particular company or debt issued by the company." LO 14.b

It is not uncommon for one company to attempt to take over another by acquiring a significant percentage of its voting shares. This is called A) arbitrage. B) recapitalization. C) a tender offer. D) preemptive rights.

A) who first wrote that option. Explanation The assignment of options contracts can be done on a first-in, first-out (FIFO) basis or by any other method that the Options Clearing Corporation considers fair (i.e., random). Determining assignment by the size of contract or on a last-in, first-out basis is not considered fair. LO 10.b

It would be fair and equitable for a brokerage firm to assign an option exercise notice to a customer A) who first wrote that option. B) with the smallest open position in that option. C) who last wrote that option. D) with the largest open position in that option.

D) could jeopardize their limited partner status. Explanation If limited partners—either individually or as a group—become too involved with the business of the partnership, they could be considered general partners and lose their limited liability. LO 11.d

Limited partners assisting the general partner to solicit new investors A) is permitted if stated in the partnership agreement. B) is permitted if done within 90 days of her acceptance as limited partner. C) is permitted if no compensation is paid. D) could jeopardize their limited partner status.

D) put a temporary hold on releasing the funds for not longer than 15 business days. Explanation FINRA Rule 2165 permits a member firm that has a reasonable belief that financial exploitation may be occurring to place a temporary hold on the disbursement of funds or securities from the account. Temporary means not longer than 15 business days. LO 1.d

Jack Mercure, age 72, has been a client of yours for many years. You have noticed he's a bit slower than before, but nothing troublesome. This morning, you get a call from Jack, and he wants you to wire a relatively substantial sum from his account to someone with a foreign address. Fearing this might be a case of senior exploitation, you discuss this with your manager. If the feeling is mutual, FINRA rules would permit your firm to A) report this situation to the Office of Foreign Assets Control (OFAC). B) freeze the client's account for not longer than 15 business days. C) contact Jack's physician to inquire about any cognitive decline. D) put a temporary hold on releasing the funds for not longer than 15 business days.

D) new clothes. Explanation The Uniform Transfer to Minors Act permits the custodian to use the funds for almost anything that is a benefit to the minor. The primary exceptions are those that most states consider to be the parental obligations of food, clothing, and shelter. LO 1.b

Many parents find that opening an UTMA account for their child is not only a good way to accumulate funds for the future but also a good way for the child to gain an appreciation for investing. The account custodian may use principal as well as income generated in the account to pay for all of the following for the child except A) the latest model smartphone. B) private school. C) summer camp. D) new clothes.

A) direct participation programs. Explanation It is FINRA Rule 2310 on DPP suitability that requires documentation maintained on certain aspects of the client. Among those is that the investor has a net worth sufficient to sustain the risks of the DPP, including loss of investment. Although part of dealing with customers is obtaining a financial profile, including net worth, the other choices here do not have a specific requirement to keep records on how the firm based its suitability for the purchase. LO 11.h

Member firms are required to maintain documentation regarding suitable net worth for investors in A) direct participation programs. B) real estate investment trusts. C) variable life insurance. D) high-yield bonds.

B) A $200 gain Explanation The premium received for writing a call becomes part of the stock sales proceeds, for a total of $4,100. Because the investor bought the stock five months earlier for $3,900, she incurred a $200 capital gain (short-term). LO 10.i

On February 7, a customer buys 100 shares of LMN at $39 per share and simultaneously writes an LMN Oct 35 call option at 6. If the call is exercised on July 19, what will she report for tax purposes? A) A $600 loss B) A $200 gain C) A $200 loss D) A $600 gain

B) each annuity unit's value varies with time, but the number of annuity units is fixed Explanation During the payout period, payments are based on a fixed number of annuity units established when the contract was annuitized. The value of an annuity unit varies from month to month according to the performance of the separate account, in comparison to the assumed interest rate. LO 9.c

Once a variable annuity has been annuitized A) each annuity unit's value and the number of annuity units vary with time. B) each annuity unit's value varies with time, but the number of annuity units is fixed. C) each annuity unit's value is fixed, but the number of annuity units varies with time. D) the number of annuity units is fixed, and their value remains fixed.

C) unrealized appreciation Explanation The NAV of mutual funds is marked to the market daily; the increase reflects higher market prices for the securities in the fund's portfolio. LO 8.c

One of your clients purchased shares of the Ajax Mutual Fund several months ago. At that time, the net asset value (NAV) of the fund was $17.20. Today, the NAV is $17.56, and your client wants to know what accounts for the difference. You should advise her that the difference likely represents A) realized appreciation. B) capital losses. C) unrealized appreciation. D) capital gains.

C) unrealized appreciation Explanation The NAV of mutual funds is marked to the market daily; the increase reflects higher market prices for the securities in the fund's portfolio. LO 8.c

One of your clients purchased shares of the Ajax Mutual Fund several months ago. At that time, the net asset value (NAV) of the fund was $17.20. Today, the NAV is $17.56, and your client wants to know what accounts for the difference. You should advise her that the difference likely represents A) realized appreciation. B) capital losses. C) unrealized appreciation. D) capital gains.

A) buy 4 MMLJ puts Explanation The customer is long the stock and so needs protection should the price of the stock fall, and a long put could give such protection. Buying a put on a long position is the strategy known as a protective put. Because the customer is long 400 shares, they need four puts to hedge the position. If the customer had been short the stock, they would use long calls to hedge. LO 10.d

One of your customers is long 400 shares of MMLJ common stock. The price of the stock has risen sharply since the customer purchased it. The customer is concerned about a price pullback and asks you to recommend a hedging strategy. You recommend that the client A) buy 4 MMLJ puts. B) buy 1 MMLJ call. C) buy 4 MMLJ calls. D) buy 1 MMLJ put.

D) protecting an existing long stock position. Explanation Buying a call will not protect a long stock position because a long call is a bullish strategy just as is a long position in a stock. Protection comes from buying an option with the opposite strategy, such as a long put. Buying a call is the strongest upside protection for a short stock position. A long call allows an investor to defer buying a stock by locking in the purchase price for up to nine months. Buying calls instead of buying their underlying stocks is also a strong diversification tool because it allows the investor to control many stocks in many different industries with a small outlay of funds. LO 10.d

One of your customers might buy a call option for all of the following reasons except A) deferring a stock buying decision. B) protecting an existing short stock position. C) diversifying an investor's holdings. D) protecting an existing long stock position.

C) The investor's income statement Explanation When it comes to the ability to make ongoing contributions to an investment program, the income statement is usually going to be the most reliable tool for verification. It is from the income statement that discretionary income (the amount left over after paying expenses) is determined. The balance sheet indicates any lump sum availability. The savings account is a part of the balance sheet. Objectives are important, but they are not a financial measurement. LO 2.a

One of your customers would like to begin an investment program calling for regular monthly contributions of $200. Which of the following would be the best source for determining if this plan is reasonable? A) The investor's objectives B) The investor's savings account C) The investor's income statement D) The investor's balance sheet

C) Strategic asset allocation Explanation The most appropriate asset allocation strategy for a buy and hold investor (long-term passive investor) would be strategic asset allocation. Using this strategy, the portfolio would be subject to periodic rebalancing, but rebalancing is not an asset allocation strategy. Tactical asset allocation is a suitable allocation strategy for active investors who are trying to time the market, buying when prices are down and selling when prices are up. Alpha measures an investment's actual return to the expected return; it is not an allocation strategy. LO 13.a

One of your more conservative clients is a buy and hold investor. As her registered representative, what would be the most suitable asset allocation strategy you should employ? A) Portfolio rebalancing B) Maximizing alpha C) Strategic asset allocation D) Tactical asset allocation

D) tax free. Explanation When buying an OID municipal bond, the discount must be accreted each year and treated as interest income. Because interest income from a municipal bond is tax free at the federal level, the discount is not taxed if the bond is held to maturity. If the customer had purchased at a discount in the secondary market, the discount would have been accreted and taxed as ordinary income. LO 6.f

Several years ago, one of your customers bought an original issue discount (OID) municipal bond at $960. The bond has now matured. For federal income tax purposes, the discount is A) taxed each year as ordinary income. B) taxed as a long-term capital gain. C) taxed at maturity as ordinary income. D) tax free.

D) Yields of municipal revenue bonds with 30 years to maturity Explanation The Bond Buyer's revenue index is an average yield of 25 revenue bonds with 30 years to maturity. LO 13.f

The Bond Buyer's revenue index is which of the following? A) A daily composite average of municipal revenue bond yields B) A daily balance of municipal revenue bond prices C) Yields of municipal revenue bonds with 20 years to maturity D) Yields of municipal revenue bonds with 30 years to maturity

D) decrease to a premium bond's cost basis. Explanation Premiums are amortized; discounts are accreted. For municipal bonds bought at a premium, the bondholder must adjust cost basis annually in such an amount that, if held to maturity, there is no reported capital gain or loss. The amortization is straight line, or the same amount must be amortized each year. LO 6.e

The IRS requires a municipal bondholder to use straight-line amortization for the purpose of determining the annual A) increase to a premium bond's cost basis. B) increase to a discount bond's cost basis. C) decrease to a discount bond's cost basis. D) decrease to a premium bond's cost basis.

B) the information to be provided by municipal issuers. Explanation The MSRB does not regulate issuers. Rather, it regulates the underwriting of municipal securities and subsequent secondary market trading. Disclosure requirements for issuers are mandated by the SEC. LO 6.h

The Municipal Securities Rulemaking Board (MSRB) is authorized to adopt rules concerning all of the following except A) the form and content of price quotations. B) the information to be provided by municipal issuers. C) the regulation of municipal securities advertising. D) the sale of new issues to related portfolios.

B) does not enforce its own rules Explanation The MSRB writes the rules for the municipal bond market, but is not an enforcement authority. The rules are enforced by other regulatory bodies depending on who the bond dealer making the trade is. If it was completed by a community bank, the FDIC is the enforcement body. If it is done by a member bank of the Federal Reserve, the regulator is the Federal Reserve Board (FRB). If the trade is made through a FINRA member firm, then FINRA is the enforcer. The SEC has regulatory authority over the MSRB and all other self-regulatory organizations (SROs) not the IRS. LO 6.h

The Municipal Securities Rulemaking Board (MSRB) writes the rules in the municipal bond market. The MSRB A) is the enforcement body in the municipal bond market. B) does not enforce its own rules. C) must comply with IRS regulations. D) is not regulated as an SRO by the SEC.

D) the issuer and the trustee acting for the bondholders. Explanation The bond resolution describes not only the characteristics of the proposed offering, but also the obligations the issuer has to its bondholders. LO 6.a

The bond resolution includes all covenants between A) the issuer and the Municipal Securities Rulemaking Board. B) the issuer and the bond counsel. C) the bond counsel and the bondholders. D) the issuer and the trustee acting for the bondholders.

D) cost of stock less premiums. Explanation The breakeven point for an investor who owns the underlying stock and writes a call is the cost of that stock less the premium received from the sale of the call. LO 10.h

The breakeven point for covered call writers is A) strike price plus premiums. B) strike price less premiums. C) cost of stock plus premiums. D) cost of stock less premiums.

A) par and the call price Explanation The call premium represents the difference between the call price and par. The farther away a call date, the lower the call premium. LO 6.c

The call premium on a municipal bond trading above par is best described as the difference between A) par and the call price. B) the market price and the call price. C) the amortized premium and the annual interest. D) the market price and par.

D) 7.1%. Explanation A bond with a coupon rate of 7.5% pays $75 of interest annually. Current yield equals annual interest amount divided by bond market price, or $75 / $1,055 = 7.109% or approximately 7.1%. LO 4.e

The current yield on a bond with a coupon rate of 7.5% currently selling at 105½ is approximately A) 6.5%. B) 8%. C) 7.5%. D) 7.1%.

B) purchase FHA-insured, VA-guaranteed, and conventional mortgages Explanation The FNMA buys FHA, VA, and conventional mortgages and uses them to back the issuance of debt securities. FNMA currently issues debentures, mortgage-backed securities, and certificates. LO 7.c

The function of the Federal National Mortgage Association (FNMA) is to A) issue conventional mortgages. B) purchase FHA-insured, VA-guaranteed, and conventional mortgages. C) guarantee the timely payment of interest and principal on FHA and VA mortgages. D) provide financing for government-assisted housing.

D) Industrial development revenue bonds Explanation Industrial revenue bonds, sometimes called industrial development bonds, may be nonpublic purpose bonds, and the proceeds are used to benefit private corporations. As such, the interest income from these bonds is a tax preference item in the alternative minimum tax calculation. LO 6.b

The interest from which of the following bonds might be included in the alternative minimum tax calculation? A) Special assessment bonds B) General obligation bonds C) Tax anticipation notes D) Industrial development revenue bonds

A) selling broad index calls. Explanation The only way to generate income through the use of options is to sell them. If concerned that the market may fall, selling calls is the appropriate strategy. LO 10.g

The manager of a portfolio that consists predominately of large- and mid-cap stocks could hedge against a market downturn and generate additional income by A) selling broad index calls. B) buying broad index calls. C) buying broad index puts. D) selling broad index puts.

A) Is 3 points in-the-money Explanation If an option has intrinsic value, it is in-the-money. Puts are in-the-money when the market price of the underlying asset is below the exercise price. The difference between the 35 strike and the 32 current market value represents 3 points of intrinsic value. Intrinsic value (the in-the-money amount) ignores the premium. However, the fact that the premium exceeds the intrinsic value by one point represents one point of time value. LO 10.c

The price of DFEC common stock is $32 per share. Your customer owns one DFEC Sep 35 put purchased for a premium of 4. The option A) is 3 points in-the-money. B) is 3 points out-of-the-money. C) has no time value. D) is 1 point out-of-the-money.

C) Long-term bonds Explanation Long-term debt prices fluctuate more than short-term debt prices as interest rates rise and fall. LO 4.e

The price of which of the following will fluctuate most with a change in interest rates? A) Common stock B) Short-term bonds C) Long-term bonds D) Money market instruments

C) Public utilities Explanation Utilities are defensive industries; they tend to pay dividends consistently. LO 8.g

The primary objective of a particular mutual fund is the payment of dividends, regardless of the market's current state. Capital growth is a secondary objective. Which of the following industry groups would be appropriate for the fund's portfolio? A) Aerospace B) Consumer appliances C) Public utilities D) Computer technology

A) protect employees from the mishandling of retirement funds by corporations and unions Explanation ERISA was created to protect the retirement funds of union members and employees of large corporations. ERISA guidelines state that all qualified retirement plans must be in writing, segregate funds from corporate or union assets, make prudent investments, report to participants annually, and not be discriminatory. All of these activities are audited under ERISA. LO 1.i

The primary purpose for creating ERISA was to A) protect employees from the mishandling of retirement funds by corporations and unions. B) establish a means for self-employed persons to provide for their own retirement. C) promote a retirement fund for government employees. D) provide all employees, both government and nongovernment, with an additional source of retirement income in the event that the Social Security system defaults.

C) private sector organizations only Explanation ERISA was established to protect the retirement funds of employees working in the private sector only. It does not apply to employees of public sector entities such as city and state governments. LO 1.i

The requirements of ERISA apply to pension plans established by A) municipal governments. B) both public and private sector organizations. C) private sector organizations only. D) public entities only.

B) entering more transactions than necessary, solely for the purpose of generating commissions Explanation Unnecessary transactions entered into for the purpose of generating commissions constitute churning. A charge of churning can result from both an excessive number and the excessive size of transactions. LO 1.a

The term churning refers to A) repeatedly purchasing stock to keep the price up. B) entering more transactions than necessary, solely for the purpose of generating commissions. C) repeatedly selling a stock short to prevent a price rise. D) purchasing calls on a particular stock for your own account before entering a large customer order for the stock.

C) convertible bonds Explanation The working capital of a corporation is equal to its current assets minus its current liabilities. A current liability is payable within 12 months. Because convertible bonds are long-term (not short-term) liabilities, they are not included as working capital. LO 13.d

The working capital of a corporation includes all of the following except A) accounts receivable. B) marketable securities of other companies. C) convertible bonds. D) cash.

A) III, II, IV, I Explanation Because the maturity and coupon rates are all the same, we can rank the bonds by rating. Based on the ratings given, the highest-quality bond is the Texas Telecom, rated AAA, followed in order by the bonds rated AA+, AA-, and A1. LO 4.f

Two conservative customers in their 50s are interested in preserving principal and high-current income from their investments. From first to last, in which order are the following bonds ranked in meeting your customer's needs? Fort Worth Gas 5¼s of 35, rated A1 San Antonio Transit 5¼s of 35, rated AA+ Texas Telecom 5¼s of 35, rated AAA Dallas Electric 5¼s of 35, rated AA- A) III, II, IV, I B) I, II, III, IV C) III, IV, II, I D) IV, III, I, II

B) CDOs always represent pools of assets that individually are very liquid, and that is why the CDOs themselves are very liquid Explanation In most cases, the assets comprising the CDO portfolio are small and individually not very liquid. Generally, individual investors would not have an opportunity to purchase these assets separately. Repackaging the assets, however, facilitates them being sold to individual investors in the secondary markets. LO 12.d

Two registered representatives are discussing a collateralized debt obligation (CDO) backed by cash flow from credit card payments. Which of their statements during the discussion is not true? A) CDOs are securitized products where pooling or repackaging of individual loans has occurred. B) CDOs always represent pools of assets that individually are very liquid, and that is why the CDOs themselves are very liquid. C) A customer would have to choose a tranche that has the right risk characteristics for him, in terms of suitability. D) CDOs are not considered suitable for all customers.

B) a registered representative of another member opening a 529 plan. Explanation FINRA requires prior written notification be made and prior written consent be received before any employee can open a brokerage account with other members or financial institutions. Exceptions include accounts where the only activity will be in 529 plans, mutual funds, or variable annuities. LO 1.d

Under FINRA's rules governing the activities of member broker-dealers, prior notification to the employing firm and prior written consent from the employing firm would be required to open a brokerage account for all of the following except A) a clerical employee of another member opening a margin account. B) a registered representative of another member opening a 529 plan. C) an officer of another member firm opening a cash account. D) a registered representative of another member opening an options account.

C) I and II Explanation MSRB rules state that a control relationship exists when a broker-dealer controls, is controlled by, or is under common control with the issuer of the security. LO 6.h

Under Municipal Securities Rulemaking Board (MSRB) rules regarding municipal securities, a control relationship exists when officers or employees of a broker-dealer hold positions of authority over the municipal issuer. officials of the municipal issuer hold policymaking positions at the broker-dealer. the municipal issuer is a public finance client of the broker-dealer. an employee of the broker-dealer lives in the issuer's municipality. A) III and IV B) I and III C) I and II D) II and III

A) must promptly make public disclosure of the same information. Explanation Whenever an issuer unintentionally discloses nonpublic, material information to any person outside of the issuer, the issuer must promptly make public disclosure of the same information. "Promptly" means as soon as practical—more specifically, no later than 24 hours after known by a senior officer, or the start of the next day's trading on the New York Stock Exchange, whichever is later. The press release by the end of the following day would likely be too late. It is unlikely that the exam will ask anything about Form 8-K, but when required, that form has a 4-day filing requirement. LO 14.b

Under Regulation FD, if an officer of an issuer makes an unintentional disclosure of nonpublic, material information to an institutional investor, the issuer A) must promptly make public disclosure of the same information. B) must file Form 8-K at or prior to month end. C) must prepare and distribute a press release by the close of business on the following day. D) is not required to take any specific action because the disclosure was unintentional.

B) the blotter or any record of original entry Explanation The six-year records include blotters, the general ledger, the stock record, customer ledgers, and customer account information. LO 15.e

Under SEC and FINRA recordkeeping rules, all of the following records must be retained for three years only except A) the subsidiary ledgers. B) the blotter or any record of original entry. C) a trial balance. D) order tickets.

C) computer hardware. Explanation Soft dollars is a term used to describe payments made by broker-dealers to investment advisers in return for research and other eligible services. The difference between soft dollars and hard dollars (cash) is that instead of paying a broker-dealer with cash, the fund will pay with brokerage business. Soft dollars may be used to pay for research, software, services for the benefit of clients, and seminar registration fees. Not permitted by the SEC are computer hardware, office equipment, and reimbursement of travel expenses to attend seminars. LO 14.d

Under SEC rules, soft dollars may be used to pay for all of the following except A) research reports. B) computer software. C) computer hardware. D) seminar registration fees.

D) II and III Explanation Any change in a customer's status, particularly those that may affect the suitability of recommendations, requires a broker-dealer to update the customer account record and furnish it to the customer within 30 days of receipt of the change notice. LO 15.c

Under SEC rules, which of the following events require a broker-dealer to furnish a copy of the account record to a customer? Change of broker-dealer's address Change of customer's name or address Change of customer's investment objectives Change in registered representative assigned to the account A) I and III B) II and IV C) I and IV D) II and III Explanation Any change in a customer's status, particularly those that may affect the suitability of recommendations, requires a broker-dealer to update the customer account record and furnish it to the customer within 30 days of receipt of the change notice. LO 15.c

D) The conversion of convertible bonds into common stocks Explanation Dilution of equity occurs when stockholders experience a reduction in their percentage ownership of the company. If bonds are converted, more common shares are issued, and the shareholder's equity is diluted. A stock dividend or stock split does not change a stockholder's percentage of ownership. Refunding debts has no effect on stockholders. LO 5.c

Under what circumstances will a dilution of equity occur? A) Issue of mortgage bonds to replace debentures B) Stock dividend C) Stock split D) The conversion of convertible bonds into common stocks

C) interest rate Explanation A variable-rate bond is one whose coupon is adjusted periodically (semiannually or annually) to reflect current interest rates. Therefore, if rates rise and force prices down, the coupon on a variable-rate bond will be adjusted upward, thereby tending to keep the bond's price at or near par. Therefore, no interest rate risk is associated with these bonds. However, if rates fall, the coupon will be adjusted downward, keeping the bond's price at or around par. Normally, a fall in rates will force prices up, but not with variable-rate bonds. LO 6.b

Variable-rate municipal bonds are subject to all of the following risks except A) market. B) liquidity. C) interest rate. D) default.

A) Appreciation potential Explanation Appreciation potential is generally not associated with existing real estate programs because most appreciation occurs in the earliest years for real estate assets. LO 11.e

Which of the following is not generally associated with an existing real estate direct participation program? A) Appreciation potential B) Lower risk than other types of real estate programs C) Immediate income stream D) Known history of income and expenses

D) An investor is not liable for taxes if he automatically reinvests the distributions. Explanation Mutual funds pay dividends from net investment income, and shareholders are liable for taxes on all distributions, whether reinvested or taken in cash. Dividend distributions from mutual funds like individual corporate dividend distributions are not guaranteed. LO 8.f

Wanting to be clear about an upcoming mutual fund dividend distribution, a shareholder does some research on his own. He comes away with what he thinks are all correct understandings, but one of them is not. Which understanding is not correct? A) The dividends are paid from the fund's net investment income. B) An investor can receive dividends in the form of cash or choose to reinvest them in more fund shares. C) Dividend distributions are not guaranteed to occur annually, quarterly, or at any time. D) An investor is not liable for taxes if he automatically reinvests the distributions.

B) Write a put on the stock Explanation Normally, investors who are short a stock, buy calls on that stock to protect the upside from unlimited potential loss. This question does not deal with that situation. Although the investor is short stock, the objective here is generating income. Selling (writing) an option is the only way to do that. Of the two choices, writing the put is the more logical. Short sellers of stock have an obligation to buy the stock to cover the short position. Writing the put generates income while also obligating the writer to buy the stock at the strike price if the holder of the option exercises. The stock put to the writer can be used to cover the short stock position, and everything is closed out. If you chose "write a call," a call writer is obligated to sell stock at the exercise price, stock the investor does not own. This investor is already obligated to buy back the stock sold short; why create an obligation to go into the market to buy more. In fact, writing a call would expose this investor to unlimited potential loss on both the short stock position and the uncovered call. LO 10.h Question #4 of 10 Question ID: 1264884

What option strategy might be used by an investor with a short position in a stock who wishes to generate some income? A) Write a call on the stock B) Write a put on the stock C) Go long a call on the stock D) Go long a put on the stock

D) Leverage is increased. Explanation Leverage is the use of someone else's money at a fixed cost to benefit the common shareholders. Issuing additional bonds increases the company's debt (money borrowed from someone else), and therefore, increases leverage for shareholders. LO 13.d

When a company issues additional bonds, which of the following is true? A) Leverage is not affected when debt securities are issued. B) It cannot be determined by only knowing that additional bonds have been issued. C) Leverage is decreased. D) Leverage is increased.

C) the customer's name and deliver the securities to the customer Explanation The term transfer and ship means to transfer the securities into the name of the customer and ship (deliver) the securities to the customer. To hold in street name would require the securities to be transferred into the name of the broker-dealer and held for safekeeping. LO 1.a

When a customer instructs a registered representative to transfer and ship, the representative instructs the margin department to transfer ownership into A) the brokerage firm's name and deliver the securities to the customer. B) the customer's name and deliver the securities to the customer's bank for safekeeping. C) the customer's name and deliver the securities to the customer. D) the brokerage firm's name and deliver the securities to the brokerage firm's commercial bank for safekeeping.

B) All gains and losses are recognized on the transfer date. Explanation Although a transfer within a family of funds is generally not subject to a sales charge, the customer is liable for any taxes due. The IRS considers this transaction a sale and a purchase. Any losses or gains must be declared on that year's tax form. LO 8.e

When a customer transfers the proceeds of a sale from one fund to another within the same family of funds, what are the tax consequences? A) Gains are taxed at the time of the transfer, but losses are deferred until the final redemption. B) All gains and losses are recognized on the transfer date. C) Losses are deducted at the time of the transfer, but gains are deferred until the final redemption. D) No gains or losses are recognized until the final redemption.

C) long-term capital gains Explanation When calculating NII, capital gains are not included. It is solely income from dividends and interest that make up the gross investment income. NII is the gross investment income minus the expenses. A regulated investment company is required to distribute a minimum of 90% of its NII to shareholders if it wishes the special tax treatment offered by the IRS. This requirement is also true when it comes to distributing long-term capital gains. The treatment of short-term gains is beyond the scope of the exam. LO 8.f

When a mutual fund computes its net investment income, all of the following are included except A) interest. B) dividends. C) long-term capital gains. D) expenses.

C) II, I, IV, III Explanation The steps in opening a new options account occur in the following order: obtain essential facts about the customer, have the manager approve the account, enter the initial order, and have the customer sign the options agreement within 15 days. What about delivery of the options disclosure document (ODD)? That isn't included in the choices here. It must be delivered at or prior to the time the account is approved for options trading. That would mean before or simultaneously with choice I. LO 10.j

When a registered representative opens an options account for a new client, in which order must the following actions take place? Obtain approval from the branch manager Obtain essential facts from the customer Obtain a signed options agreement Enter the initial order A) II, I, III, IV B) I, II, III, IV C) II, I, IV, III D) I, II, IV, III

B) The intended use of funds raised by the issue Explanation The customer's state of residence and tax bracket are important because these factors help establish the tax benefits offered by the municipal bond. The bond's rating is important in evaluating the credit risk assumed by the investor. While the intended use of the funds may be of interest to the customer, it would be of little consideration when making a purchase recommendation. LO 6.c

When a registered representative recommends a municipal bond purchase to a customer, which of the following would be of least consideration regarding suitability? A) The bond's rating B) The intended use of funds raised by the issue C) The customer's state of residence D) The customer's tax bracket

A) a waiver of premium option Explanation A waiver of premium option allows the premiums on an insurance contract to be waived for a person who has become disabled or otherwise unable to work. LO 13.g

When an insured person becomes disabled or unable to work, that person may not be required to continue paying premiums on the contract if the contract included A) a waiver of premium option. B) a freeriding provision. C) a disability rider. D) a low-income provision.

A) securitization Explanation Asset-backed securities represent a pool of assets that were combined into a financial instrument such as a CMO for the purpose of better facilitating sales to the general investing public. The process of pooling assets into a single financial instrument for this purpose is known as securitization. LO 12.c

When assets are pooled into financial instruments such as collateralized mortgage obligations (CMOs) to better facilitate selling them to the general public, the process is known as A) securitization. B) best efforts. C) structuring. D) diversification.

A) II and IV Explanation Contracts on each side of the market are used for determining position limits. Long calls and short puts are on the same side (bullish), and long puts and short calls are on the same side (bearish). LO 10.j

When determining a position limit, a member firm aggregates which of the following customer positions? Long calls and long puts Long calls and short puts Short calls and short puts Short calls and long puts A) II and IV B) I and III C) I and IV D) II and III

A) The contracts must be aggregated Explanation LEAPS and listed options on the same side of the market, on the same underlying security, must be aggregated and remain within position limits. LO 10.j

When determining position limits for listed options contracts and LEAPS contracts on the same side of the market, which of the following statements is true? A) The contracts must be aggregated. B) The contracts do not have position limits. C) The contracts are considered separately. D) The contracts are added to increase the position limits.

D) Principal amount Explanation In judging whether bonds purchased are substantially identical to bonds sold for a loss, the tax code considers maturity, issuer, and coupon rate. If at least two of the three are different, a wash sale will generally not result. LO 13.h

When determining whether a tax swap of municipal bonds will result in a wash sale, which of the following is not considered? A) Coupon B) Maturity C) Issuer D) Principal amount

B) geographical diversification. Explanation Diversification is a proven way of mitigating certain risks. A popular method in the municipal bond industry is to diversify by purchasing bonds issued in different parts of the country. There are other forms of diversification, but none of those are employed by this client. The coupon rates are all about the same. The quality is all about the same, as are the maturities (duration). LO 13.a

When examining the portfolio of a municipal bond client, you notice that every bond is Aaa or Aa rated. In addition, each bond matures at about the same time and the coupon rates do not vary by more than 50 basis points. You also notice that over 10 different states are represented by the issuers with no state being represented more than twice. It would appear that this client has used A) duration diversification. B) geographical diversification. C) interest rate diversification. D) quality diversification.

A) At or before the time the account receives approval for options trading Explanation Customers must receive the OCC Disclosure Booklet at or before the time their account is approved for options trading. LO 10.j

When must a new options customer—who has not yet traded options—receive the Options Clearing Corporation's (OCC's) current disclosure document? A) At or before the time the account receives approval for options trading B) No later than 15 days after the ROP signs the options customer approval form. C) At or before the time the registered representative signs the customer approval form D) Within 15 days of the ROP's approval of the customer's account for options trading

A) the options disclosure document (ODD) Explanation Any prospective or new options customer must receive a copy of a booklet titled "Characteristics and Risks of Standardized Options." In every day usage (and on the exam), it is referred to as the options disclosure document (ODD). It serves the purpose of a prospectus and discloses the risks of investing in options. This is another case where you must select the most accurate choice. LO 2.g

When opening an options account, the customer must be provided with A) the options disclosure document (ODD). B) the options prospectus. C) the options arbitration agreement. D) the options risk disclosure document.

C) common stock Explanation Money market instruments are short-term, high-quality debt securities. This includes treasuries with less than one year to maturity and negotiable CDs. Because common stock is equity, it is not found in money market funds. LO 8.g

When reviewing a money market fund portfolio, one would not expect to find A) T-bills. B) negotiable CDs. C) common stock. D) T-bonds with less than one year to maturity.

D) Ginnie Maes Explanation Direct debt is backed in full by the U.S. government. The Government National Mortgage Association is owned by the U.S. government; thus, Ginnie Maes are fully backed. Treasury receipts are zero-coupon bonds based on U.S. government debt instruments but are created and issued by broker-dealers and, as such, are not direct obligations of the U.S. government. LO 7.c

Which of the following are a direct obligation of the U.S. government? A) Fannie Maes B) Treasury receipts C) Government bond mutual funds D) Ginnie Maes

B) High inflation risk, low default risk Explanation A longer term bond will be subject to more inflation risk. Because the quality of the bond is high, the level of default risk should be low. LO 14.a

Which of the following best describes the investment characteristics of a high-quality long-term municipal bond? A) Low inflation risk, high market risk B) High inflation risk, low default risk C) Low inflation risk, low default risk D) High inflation risk, high market risk

C) Blotters and general ledgers Explanation Five main brokerage records must be retained for six years: blotters, general ledgers, customer ledgers, stock records, and customer account records. Articles of incorporation are lifetime and the other two choices have a 3-year retention requirement. LO 15.e

Which of the following brokerage records must be kept for six years? A) Bank statements and trial balances B) Articles of incorporation or partnership agreements C) Blotters and general ledgers D) Customer correspondence and order tickets

A) A sole proprietorship Explanation A sole proprietorship has only one owner. That is what the "sole" means. Although S corporations and LLCs can be formed with a single shareholder or member, that would be the exception rather than the rule. A partnership needs at least two persons. LO 1.c

Which of the following business structures will generally have the fewest number of owners? A) A sole proprietorship B) An S corporation C) An LLC D) A general partnership

D) Financial Guaranty Insurance Corp. Explanation Standard & Poor's, Fitch, and Moody's are all agencies that rate debt securities, including municipals and equity securities. The Financial Guaranty Insurance Corp. is one of several entities that insure municipal bonds. LO 4.f

Which of the following choices is least similar to the others? A) Standard & Poor's. B) Moody's. C) Fitch. D) Financial Guaranty Insurance Corp.

D) Collateral trust bond Explanation Collateral trust bonds are backed by a portfolio of other securities, while mortgage bonds are backed by real estate. Equipment trust certificates are backed by equipment, while debentures are backed only by the company's promise to pay. LO 5.a

Which of the following corporate bonds is backed by other securities? A) Debenture B) Mortgage bond C) Equipment trust certificate D) Collateral trust bond

A) AAA/AAA-rated debenturesExplanation Corporate debentures are unsecured bonds backed by the credit of the issuing corporation; they are not secured by underlying collateral. Mortgage bonds are secured with real estate serving as collateral. Collateral trust bonds are secured by securities that a corporation owns in other companies or bonds. Equipment trust certificates are secured by transportation equipment owned by the corporation. LO 5.a

Which of the following debt instruments is unsecured? A) AAA/AAA-rated debentures B) Junior lien mortgage bonds C) Equipment trust certificates D) Collateral trust certificates

D) Collateralized mortgage obligation (CMO) Explanation CMOs are mortgage-backed securities. Because mortgages are often paid off ahead of the scheduled maturity, the exact maturity date of a CMO is uncertain. LO 12.d

Which of the following debt securities does not have a fixed maturity date? A) General obligation bond B) Treasury STRIPS C) Subordinated debenture D) Collateralized mortgage obligation (CMO)

D) Options Clearing Corporation (OCC) Explanation The OCC guarantees the performance of listed option contracts. LO 10.b Explanation The OCC guarantees the performance of listed option contracts. LO 10.b

Which of the following entities guarantees a listed yield-based option? A) Broker-dealer B) U.S. government C) Federal Reserve Board (FRB) D) Options Clearing Corporation (OCC)

C) The stock's price recently penetrated its resistance level Explanation Technical analysts focus on market price patterns. Typically, a technical analyst becomes bullish when a stock's price exceeds its resistance level (it penetrates its former ceiling price). Book values, credit ratings, and the resignation of a company's senior officer are of interest to fundamental analysts. LO 13.e

Which of the following events is of the greatest importance to a technical analyst? A) The issuer's current book value increased. B) The company announced the resignation of its chief financial officer. C) The stock's price recently penetrated its resistance level. D) Standard & Poor's raised the credit rating of the issuer.

C) The stock's price recently penetrated its resistance level Explanation Technical analysts focus on market price patterns. Typically, a technical analyst becomes bullish when a stock's price exceeds its resistance level (it penetrates its former ceiling price). Book values, credit ratings, and the resignation of a company's senior officer are of interest to fundamental analysts. LO 13.e

Which of the following events is of the greatest importance to a technical analyst? A) The issuer's current book value increased. B) The company announced the resignation of its chief financial officer. C) The stock's price recently penetrated its resistance level. D) Standard & Poor's raised the credit rating of the issuer.

C) Reimbursement for meal expenses incurred while attending an investment seminar Explanation Payment for travel expenses, furniture, or equipment is not allowable under Section 28(e) of the Securities Exchange Act of 1934. Payment for seminars, research, and financial planning software are permissible under the safe harbor provisions of Section 28(e). LO 14.d

Which of the following forms of soft-dollar compensation paid by a broker-dealer to an investment adviser is not allowable under the safe harbor provisions of Section 28(e)? A) Registration fees to attend an investment seminar B) Financial planning software C) Reimbursement for meal expenses incurred while attending an investment seminar D) Research reports

B) An individual with current income consisting of dividends and capital gains only Explanation An IRA contribution may be made only from earned income. While dividends and interest are investment income, alimony received as part of a divorce settlement entered into prior to January 1, 2019, is considered compensation for IRA purposes. Individuals may contribute to an IRA even if they are already covered by a corporate pension plan or Keogh plan. Although a contribution can be made, it may or may not be deductible, depending on the individual's income. LO 1.g

Which of the following individuals would not be permitted to contribute to an IRA? A) A self-employed attorney who already has a Keogh plan established B) An individual with current income consisting of dividends and capital gains only C) A corporate officer who is covered by a company-sponsored 401(k) plan D) An individual divorced on December 26, 2018,whose sole income is alimony and child support from a former spouse

A) Zero-coupon bonds Explanation Reinvestment risk to interest is the uncertainty that periodic payments from an investment, such as bond interest, can be reinvested at the same rate as investment is currently paying without increasing the risk. Because the zero-coupon bond makes no periodic interest payments, there is nothing to reinvest. It makes no difference how high the coupon yield is or how secure the interest payments are, the investor never knows what current market interest rates will be when the semiannual checks arrive. LO 14.a

Which of the following investments is likely to have the least reinvestment risk to interest? A) Zero-coupon bonds B) High-yield bonds C) U.S Treasury bonds D) Double-barreled bonds

B) A covered call (selling a call option on a stock owned within the IRA) Explanation Short sales, uncovered calls, and municipal bonds are all inappropriate for individual retirement accounts. Covered calls are allowed and will be covered in detail in Unit 4. LO 1.g

Which of the following investments would be most suitable for an IRA? A) Highly rated general obligation bond B) A covered call (selling a call option on a stock owned within the IRA) C) Uncovered call on a stock whose price is extremely stable D) Short sale of a stock that has just started what is expected to be a prolonged decrease in price

B) Bridge authority revenue bonds guaranteed by the full faith and credit of a city Explanation Double-barreled bonds are backed not only by a specified source of revenues, but also by the full faith and credit of a municipal issuer with authority to levy taxes. Even though they are rated and traded as if they were general obligation bonds, it is proper to include them in the revenue category because the initial backing is from revenue. The additional backing of PHAs is the full faith and credit of the U.S. government—not the issuer. The additional backing of moral obligation bonds are legislative appropriations, which are not mandatory. LO 6.b

Which of the following is considered a double-barreled bond? A) Build America Bonds B) Bridge authority revenue bonds guaranteed by the full faith and credit of a city C) Moral obligation bonds D) Dome stadium bonds with provisions for emergency ceiling support

D) The at-risk provisions do not apply to oil and gas exploration programs. Explanation The at-risk provisions (you can only deduct what you can lose) apply to all DPPs. Real estate has one unique feature in that nonrecourse financing is part of the investor's tax basis. LO 11.f

Which of the following is not a correct statement in respect to the at-risk provisions when investing in a direct participation program (DPP)? A) Qualified nonrecourse financing is excluded from tax basis except in the real estate programs. B) Losses disallowed by the at-risk provisions in any one year may be carried over to following taxable years. C) Deductions or losses are limited to the investors' invested capital plus their percentage of partnership liabilities for which they are personally liable. D) The at-risk provisions do not apply to oil and gas exploration programs.

A) PSA Explanation PSA is the Public Securities Association. What does that have to do with CMOs? Plenty. It is the PSA table that is used to project the prepayment schedule for CMO tranches. However, unlike the PAC (planned amortization class) or the TAC (targeted amortization class) and the Z (zero), it is not a tranche. LO 12.d

Which of the following is not a type of CMO tranche? A) PSA B) Z C) TAC D) PAC

B) The call has a time value beyond an intrinsic value that gradually dissipates Explanation Call options allow greater leverage than buying the underlying stock, and the capital requirements are smaller, allowing for a smaller loss potential. The fact that options expire (i.e., have a time value that erodes as the option nears expiration) is a disadvantage of options. Stock purchases have no time value component—there is no expiration and no resulting value erosion. LO 10.c

Which of the following is not an advantage of buying listed call options as compared to buying the underlying stock? A) Buying a call would require a smaller capital commitment. B) The call has a time value beyond an intrinsic value that gradually dissipates. C) Buying a call allows greater leverage than buying the underlying stock. D) Buying a call has a lower dollar-loss potential than buying the stock.

B) Treasury bonds Explanation Treasury bonds are long-term investment vehicles having maturities of 10 years or more. LO 7.b

Which of the following is not considered a short-term investment vehicle? A) Commercial paper B) Treasury bonds C) Repurchase agreements D) Negotiable CDs

B) A hedge fund Explanation Hedge funds do not meet the definition of investment company as described in the act. The three types of investment companies are the face amount certificate company, the unit investment trust, and the management investment company (closed-end and open-end). LO 12.a

Which of the following is not included in the definition of an investment company under the Investment Company Act of 1940? A) A unit investment trust B) A hedge fund C) A closed-end fund D) An open-end fund

C) Defined benefit Explanation In a defined benefit plan, the payout is established, and employers must contribute annually to assure payment of the benefit amount. An actuary must calculate the annual contribution amount necessary to meet the benefit requirement. LO 1.h

Which of the following plans requires an actuary's services? A) 401(k) B) Profit-sharing C) Defined benefit D) Defined contribution

A) I and II The expiration dates and strike prices may be different or the same. However, the total number of contracts on the same side of the market is limited to 100,000 for this stock. Long calls and short puts are on the same side of the market (the bull side), and short calls and long puts are on the same side of the market (the bear side). LO 10.j

Which of the following positions violate the rules governing position limits? (Assume SSS stock is subject to a 100,000-option position limit.) Long 50,000 SSS Aug 40 calls, short 55,000 SSS Aug 40 puts Long 50,000 SSS Aug 40 calls, short 55,000 SSS Jan 40 puts Long 50,000 SSS Aug 40 calls, short 30,000 SSS Jan 40 calls Long 50,000 SSS Sep 40 puts, short 45,000 SSS Sep 40 calls A) I and II B) II and III C) I and IV D) III and IV

B) I and II Explanation The expiration dates and strike prices may be different or the same. However, the total number of contracts on the same side of the market is limited to 100,000 for this stock. Long calls and short puts are on the same side of the market (the bull side), and short calls and long puts are on the same side of the market (the bear side). LO 10.j

Which of the following positions violate the rules governing position limits? (Assume SSS stock is subject to a 100,000-option position limit.) Long 50,000 SSS Aug 40 calls, short 55,000 SSS Aug 40 puts Long 50,000 SSS Aug 40 calls, short 55,000 SSS Jan 40 puts Long 50,000 SSS Aug 40 calls, short 30,000 SSS Jan 40 calls Long 50,000 SSS Sep 40 puts, short 45,000 SSS Sep 40 calls A) III and IV B) I and II C) II and III D) I and IV

C) A new high school Explanation Hospitals, airports, and golf courses all generate revenue and can be financed with revenue bond issues. Schools are financed through general obligation bond sales. LO 6.b

Which of the following projects is most likely to be financed by a general obligation rather than a revenue bond? A) A public golf course B) An expansion of an airport C) A new high school D) A municipal hospital

D) Minutes of the board of directors' meetings Explanation The stock certificate book, articles of incorporation or partnership agreement, and minutes of the directors' meetings are to be kept on file and accessible for the life of the firm. The new account form and the general ledger are six-year records, and the complaint file is the only record with a four-year retention requirement. LO 15.e

Which of the following records must kept for the life of a broker-dealer organized as a corporation? A) Complaint records B) The general ledger C) Customer new account forms D) Minutes of the board of directors' meetings

B) I and IV Explanation T-bills trade at a discount to par, are six months or less to maturity, and are a direct obligation of the U.S. government. T-bills are also noncallable. LO 7.b

Which of the following regarding T-bills are true? T-bills trade at a discount to par. T-bills have maturities of 1 to 10 years. Most T-bill issues are callable. T-bills are a direct obligation of the U.S. government. A) II and III B) I and IV C) I and III D) II and IV

C) Syndicator Explanation A syndicator handles the registration of the limited partnership units. LO 11.c

Which of the following registers the securities and packages the program for a limited partnership? A) General partner B) Property manager C) Syndicator D) Limited partners

D) TIPS bonds Explanation TIPS bonds adjust the principal value each six months based on the inflation rate. If the inflation rate is positive, the value increases. Those increases are reported as income each year even though the investor does not receive the appreciation until the bonds mature (or are sold). LO 7.a

Which of the following securities can generate phantom income? A) Treasury bonds B) Treasury bills C) Treasury notes D) TIPS bonds

D) Municipal bonds Explanation The legal opinion is supplied by bond counsel, an attorney specializing in securities law. The opinion states that the municipal bond is legal and binding on the issuer. If the bond's interest is tax-exempt, that opinion is stated as well. Legal opinions are exclusive to municipal securities. LO 6.a

Which of the following securities is typically sold with the legal opinion attached? A) Participating preferred stock B) Corporate bonds sold under the Trust Indenture Act of 1939 C) U.S. Treasury issues D) Municipal bonds

A) TANs Explanation TANs are tax anticipation notes. These are short-term municipal securities and that is what Moody's MIG ratings represent. MIG stands for Municipal Investment Grade. GOs are rated with the normal letter ratings and BAs (bankers' acceptances) and T-bills are not municipal securities. LO 6.f

Which of the following securities would have a Moody's MIG rating? A) TANs B) BAs C) GOs D) T-bills

C) I, II, and III Explanation Original issue premium municipal bonds (as well as those purchased in the secondary market) must be amortized by an amount each year so that, if held to maturity, there is no reported capital loss. LO 6.e

Which of the following statements about municipal original issue premium bonds are true? The original issue premium must be amortized. If the bond is held to maturity, there will be no capital loss reportable. The cost basis of the bond is adjusted downward by the amortized amount. A) I and II B) I and III C) I, II, and III D) II only

D) I and III Explanation State debt cannot overlap with any other municipal entity. LO 6.b

Which of the following would not be examples of overlapping debt? Debt to build a state office building within city limits Debt to maintain a county park district serving a municipality. Debt backed by two states cooperating in the construction of a bridge Debt for a high school district within city limits A) II and IV B) II and III C) I and II D) I and III

A) Money is raised without a specific property being stated, and the general partner selects the investments Explanation A blind pool offering, also known as a nonspecified program, involves an investment in a program without specific prospects or properties being identified. LO 11.g

Which of the following statements describes an oil and gas blind pool offering? A) Money is raised without a specific property being stated, and the general partner selects the investments. B) An unknown number of representatives participate in the sale of known partnership units. C) The income from producing wells is purchased at a discount from the present value of the projected future flows. D) The oil exploration occurs in an area that is not adjacent to any known oil reserves.

A) I and IV Explanation GNMA securities are subject to both state and federal income tax and are backed by residential mortgages. LO 7.c

Which of the following statements regarding Government National Mortgage Association (GNMA) securities are true? Interest is subject to federal income tax. Interest is exempt from federal income tax. They are backed by farm mortgages. They are backed by residential mortgages. A) I and IV B) II and IV C) I and III D) II and III

B) I and IV Explanation T-bills are sold at a discount and can be purchased in minimum denominations of $100. The difference between the purchase price and the maturity value is taxed as interest income, not as a capital gain. Treasury bills are short-term investments maturing in 1 year or less. T-notes have maturities of 2 to 10 years. T-bonds have maturities of longer than 10 years. LO 7.b

Which of the following statements regarding Treasury bills T-bills are true? The government auctions T-bills at a discount. The difference between the cost of a T-bill and its value at maturity is treated as a capital gain. T-bills have longer maturities than T-notes. The minimum denomination of a T-bill is $100 face amount. A) I and III B) I and IV C) II and III D) II and IV

A) I and II Explanation Because an ETF is purchased and sold on an exchange, the rules generally applying to all exchange products, such as purchasing them on margin, would apply. Leveraged funds can use a number of different securities types, including derivative products, and trading techniques, such as trading on margin, as a means of attaining the leveraged returns they promise. LO 8.h

Which of the following statements regarding a leveraged exchange-traded fund (ETF) are true? The leveraged ETF may be purchased on margin. Securities within the leveraged fund portfolio may be purchased on margin. The leveraged ETF may never be purchased on margin. Securities within the leveraged fund portfolio may never be purchased on margin. A) I and II B) III and IV C) II and III D) I and IV

C) II and IIIExplanation The investor in a corporate zero coupon bond receives the return in the form of growth of the principal amount over the bond's life. The bond is purchased at a deep discount and redeemed at par at maturity. That discount from par represents the interest that will be earned at maturity date. However, the discount is accreted annually, and the investor pays taxes yearly on the imputed interest. LO 5.a

Which of the following statements regarding corporate zero coupon bonds are true? Interest is paid semiannually. The discount is in lieu of periodic interest payments. The discount must be accreted and is taxed annually. The discount must be accreted annually with taxation deferred until maturity. A) I and III B) I and IV C) II and III D) II and IV

A) I and III Explanation With qualified plans, participants receive a tax deduction for contributions to their plan. As earnings accumulate tax deferred, distributions, which consist of tax-deferred earnings and contributions for which the participant received a tax deduction, are 100% taxable. LO 1.f

Which of the following statements regarding qualified retirement plans are true? Contributions are made with pretax dollars. Contributions are made with after-tax dollars. Distributions are 100% taxable. Distributions are taxable only to the extent of earnings. A) I and III B) I and IV C) II and III D) II and IV

A) They are direct obligations of the U.S. government. Explanation With the exception of Ginnie Mae, all agency securities are indirect obligations of the U.S. government. LO 7.c

Which of the following statements regarding the Federal Farm Credit System securities are not true? A) They are direct obligations of the U.S. government. B) Interest is tax exempt at the state and local levels. C) They issue short-term notes and long-term bonds. D) The proceeds are used to make loans to farmers.

B) In every discussion about the benefits of options, a statement must be made regarding the corresponding risks. Explanation Any description of options must include a description of the risks. This rule applies to all communications with the public—written, electronic, or in person. LO 10.j

Which of the following statements regarding the discussion of options with customers is true? A) An Options Clearing Corporation options disclosure document must be in a customer's hands before options can be discussed. B) In every discussion about the benefits of options, a statement must be made regarding the corresponding risks. C) Buying or selling straddles only would require no risk disclosures. D) Covered call writing has such a limited risk that it is unnecessary to point out risk factors.

C) I and IV

Which of the following statements regarding warrants are true? They pay dividends. They represent ownership in the issuing corporation. They allow for the purchase of common stock at a fixed price. They do not give holders voting rights. A) I and II B) III and IV C) II and IV D) II and III

C) Systematic risk Explanation Market risk, sometimes referred to as systematic risk, cannot be diversified away. The risk of investing in a single industry or sector can be diversified away by investing in several industries with returns not correlated to each other. A general downturn in the market, however, cannot be eliminated through diversification. LO 14.a

Which of the following types of risk cannot be eliminated through diversification under the modern portfolio theory? A) Business risk B) Liquidity risk C) Systematic risk D) Interest rate risk

C) An airline company Explanation When you see "equipment trust certificate," think transportation companies such as airlines and railroads. LO 5.a

Which of the following would be most likely to issue an equipment trust certificate? A) A social media company installing new servers B) A user of farming equipment C) An airline company D) A company using specialized equipment on an oil drilling rig

B) II and III only Explanation If the dollar is devalued, it becomes less valuable in foreign countries. That means that more dollars are required to purchase the same amount of foreign goods. The increased cost of those foreign goods will reduce imports of them. On the other side, because the foreign currency now goes further in the United States, goods made here become cheaper to buy, so exports will increase. LO 10.g

While watching the financial news on TV, you hear an internationally recognized economist say that she expects a significant devaluation of the U.S. dollar. If she is correct, what would be the likely effect on foreign trade? The price of foreign goods would decrease, leading to an increase in imports. The price of foreign goods would increase, leading to a decrease in imports. The price of U.S.-made goods would decrease, leading to an increase in exports. The price of U.S.-made goods would increase, leading to a decrease in exports. A) I and IV only B) II and III only C) II and IV only D) I and III only

C) neutral. Explanation This position is a short combination where both contracts are in the money. Breakeven points are 51 and 44. Above or below these points, the customer will lose money. LO 10.f

With XYZ trading at $47.50, your customer writes 1 XYZ Jan 50 put and simultaneously writes 1 XYZ Jan 45 call, receiving $600 in combined premiums. Your customer's market attitude is A) bullish. B) bearish. C) neutral. D) speculative.

B) current assets minus current liabilities. Explanation Current means cash or assets that would be exchanged for cash in the ordinary course of business in the current year. In the case of liabilities, current means maturing or falling due within the current year. The net of current assets less the current liabilities implies the company has cash availability of the remainder with which to work. LO 13.d

Working capital is A) current assets minus inventory. B) current assets minus current liabilities. C) total assets minus total liabilities. D) only the cash and equivalents.

A) $0.45 Explanation Because the stock is trading with rights (before the ex-rights date), the formula is (M ‒ S ) divided by (N + 1). Plugging the numbers in, we have ($40 ‒ $35) divided by (10 + 1) = $5.00 ÷ 11 = $0.45. LO 3.f

XYZ Corporation, whose common stock is currently selling for $40 per share, is having a rights offering. The terms of the offering require 10 rights plus $35 to subscribe to one share of stock. Compute the theoretical value of a right before the ex-rights date. A) $0.45 B) $0.55 C) $0.50 D) $3.50

B) Begin a periodic payment deferred annuity program Explanation In this case, if the client cannot supply the funds for the single premium annuity, but in all other respects, the product is suitable, then a period payment program would work. FINRA specifically warns about using mortgage equity (a home equity loan) to fund a deferred variable annuity. Although one can have both types of IRA, the annual maximum cannot exceed the current (2020) level of $6,000 total between them. Variable annuities are not marginable securities. LO 9.e

You have a 45-year-old client whose traditional IRA contributions are at the maximum. The client would like to provide a larger cushion for retirement and asks for your advice. After reviewing the client's goals, you realize that a $25,000 single premium deferred variable annuity purchase would be the ideal solution. When approaching the client with this recommendation, the client tells you that cash of that magnitude is not readily available. Which of the following would you recommend? A) Purchase the deferred variable annuity on margin B) Begin a periodic payment deferred annuity program C) Open a Roth IRA to double the amount of allowable IRA contributions D) Take a home equity loan to fund the deposit

D) private offering memorandum. Explanation Hedge funds do not register with the SEC, so there is no registration statement. However, the management does prepare an offering document. It could be called the private placement memorandum, the offering circular, or even the prospectus on the exam. The statement of additional information (SAI) is limited to registered management investment companies (open- and closed-ends) and ETFs. LO 12.a

You have a high net worth client who is interested in investing in a hedge fund. Details of the offering would be found in the fund's A) statement of additional information. B) registration statement. C) subscription agreement. D) private offering memorandum.

C) for the underlying XYZ stock, more puts than calls are being traded. Explanation The ratio is a measure of puts traded to calls traded and is calculated by dividing the number of traded puts by the number of traded calls (puts / calls). As the ratio increases, it reflects that more puts than calls are being traded and is therefore a more bearish indicator of investor sentiment. LO 10.d

Your client currently holds XYZ stock in her portfolio. You notice that the put-call ratio for options trading on XYZ stock has been increasing over the past several days. The increase in the ratio would indicate that A) for the underlying XYZ stock, straddles are being purchased. B) for the underlying XYZ stock, more calls than puts are being traded. C) for the underlying XYZ stock, more puts than calls are being traded. D) investors are becoming more and more bullish on XYZ stock.

D) cash equal to the difference between the closing value of the index and exercise price. Explanation Unlike equity options, index options are settled in cash only. Upon exercise, cash equal to the amount that the option is in the money (i.e., excess of market value over strike for a call or excess of strike over market value for a put) is delivered on the settlement date. LO 10.g

Your client with a short call position in the S&P 100 index (OEX) is assigned an exercise notice. The obligation is fulfilled by delivering A) 100 shares of a particular stock in the index selected by the option holder. B) 100 shares of each of the 100 stocks in the index. C) a long call in the OEX at the same or lower strike price. D) cash equal to the difference between the closing value of the index and exercise price.

D) $2,500 Explanation He will first offset his $2,000 in capital gains, leaving $5,500 in losses. He next offsets $3,000 in adjusted gross income, leaving $2,500 in losses to carry forward to next year. Provided the loss is offset to the maximum each year, there is no limit to how long losses may be carried forward. LO 3.i

Your customer has experienced $7,500 in capital losses this year. He has realized $2,000 in capital gains and has $65,000 adjusted gross income. How much of his loss will he be able to carry forward to next year? A) $5,500 B) None C) $4,500 D) $2,500

D) A variable annuity Explanation Variable annuities offer tax-deferred growth and are suitable for achieving supplemental retirement income. Ideally, they should be funded with readily available cash rather than using funds liquidated from existing investments. None of the other investments listed here offer tax-deferred growth. LO 9.e

Your customer in her early 30s has received a modest inheritance from a relative. Listing tax-deferred growth as an objective for retirement income, which of the following investments is most suitable? A) Growth mutual funds B) Tax-free municipal bonds C) Corporate debt securities D) A variable annuity

C) First in, first out (FIFO) Explanation When a customer does not choose a method, the IRS uses FIFO. This will likely result redeeming shares with the lowest cost basis first, which creates a greater taxable gain. LO 13.h

Your customer redeemed 200 of her 500 Kapco common shares without designating which shares were redeemed. Which of the following methods does the IRS use to determine which shares she redeemed? A) Wash sale rules B) Last in, first out (LIFO) C) First in, first out (FIFO) D) Identified shares

C) Treasury STRIPS Explanation The Treasury STRIPS is long-term, no-interim income security and has a locked-in yield because it is purchased at a discount from par. The Treasury bill is short term, the Treasury bond provides semiannual interest, and the corporate zero is riskier than the STRIPS. LO 7.a

Your customer wishes to lock in a long-term yield with minimal risk and is not interested in regular income. Which of the following securities should you recommend? A) Corporate A-rated zero-coupon bond B) Treasury bill C) Treasury STRIPS D) Treasury bond

D) Long calls and puts, long straddles, long or short spreads Explanation When purchasing calls or puts, loss is limited to the premium paid. When purchasing straddles, loss is limited to the combined premiums paid. When purchasing or selling either call or put spreads, the potential gain or loss is always defined—therefore, limited to some amount. These would be the only suitable positions offered: long calls and puts, long straddles, and long or short spreads. Each of the remaining choices contains a position with an unlimited loss potential such as short calls or short straddles. LO 10.f

Your customer with a discretionary account is approved to trade all options strategies and positions. Regarding discretion, the customer cautions that no positions having an unlimited loss potential should ever be used or would be acceptable to her. Which of the following options positions would still be suitable, and therefore, employed in the customer's discretionary account? A) Long calls and puts, short straddles B) Short spreads, short straddles, short calls and puts C) Long and short straddles, short calls and puts D) Long calls and puts, long straddles, long or short spreads

D) 7% 30-year U.S. Treasury bond Explanation Price fluctuations are the greatest in bonds with the longest terms to maturity. The riskier the instrument, the more price volatility. Long-term bonds have greater risk than short-term bonds. LO 4.e

n which of the following will a change in interest rates cause the greatest price fluctuation? A) 7% AA-rated one-year municipal note B)Series EE bond C) 7% AAA-rated corporate bond with eight years until maturity D) 7% 30-year U.S. Treasury bond


Ensembles d'études connexes

Lesson 5: The Service Value Chain

View Set

BIOL-100 A: Unit 1 - What is Biology?

View Set

Jensen's Health Assessment 3rd Ed. | Chapter 4

View Set

Chapter 10 - Pure Competition In The Long Run

View Set

Managerial Accounting Chapter 15

View Set

EXERCISE 17.2: Relative Ages in Cross-Cutting Situations

View Set